Exam 2 (Chapter 4, 5, 7, 10, 14, 17)

अब Quizwiz के साथ अपने होमवर्क और परीक्षाओं को एस करें!

12. A postpartum nurse has received an exchange report on the four following mother-baby couplets. Based on the provided information, which couplet should the nurse first assess? a. A 25-year-old G2P1 woman who is 36 hours postbirth and is having difficulty breastfeeding her baby girl. Her fundus is firm at the umbilicus, and lochia is moderate to scant. b. A 16-year-old G1P0 who will be discharged in the afternoon. It was reported that she refers to her baby boy as "it" and that she requested to have her baby stay in the nursery so she could sleep. c. A 32-year-old G5P4 woman who delivered a 4500 gram baby boy 2 hours ago after a 20 hour labor that was augmented. It was reported that her fundus is 2 cm above umbilicus with moderate lochia. d. A 28-year-old G2P1 woman who delivered a 3800 gram baby girl by elective cesarean birth. She had spinal anesthesia and was given intrathecal preservative-free morphine for postoperative pain management. Her vital signs are B/P 115/75, P 80, R 18 T 98.

ANS: C a. The priority need for this woman is breastfeeding assistance which does not require immediate attention. b. The data indicate that the woman is experiencing a delay in bonding and that social services should become involved. This needs to be done prior to discharge but does not require immediate attention. c. This woman is at risk for hemorrhage (large baby, prolonged labor, augmented labor, high parity, and immediate postpartum). This woman needs to be assessed first to determine whether the fundus is firm and if lochia is within normal limits. d. Based on data provided, this woman is stable, but should be assessed second.

Signs of possible maladaptation to pregnancy include: A. Denial of fears about childbirth and unrealistic expectations about birth B. Excessive maturity weight gain and limiting physical activity C. Denial of physical symptoms of pregnancy and emotional lability D. Changes in couple's interactions and sexual activity

A. Denial of fears about childbirth and unrealistic expectations about birth

Which of the following should be included in postpartum discharge teaching regarding risk reduction for cystitis? select all that apply A. Drink a minimum of 3,000 mL of fluid per day B. Change peripad at least every 3-4 hours C. Eat foods low in acidity D. Avoid caffeinated fluids

A. Drink a minimum of 3,000 mL of fluid per day B. Change peripad at least every 3-4 hours

A nurse is assessing a preterm baby with a gestational age of 32 weeks and birth weight of 1,389 grams. Which of the following signs if present would be a possible indication of RDS. A. Expiratory grunting and intercostal retractions. B. Respiratory rate of 46 breaths per minute and presence of acrocyanosis. C. Mild nasal flaring and heart rate of 140 beats per minute. D. Bradycardia and bounding pulse

A. Expiratory grunting and intercostal retractions

Measurable outcomes of social support interventions during pregnancy include: A. Improved coping and functional status B. Improves relationship with mother C. Reduced stress D. Reduced depression

A. Improved coping and functional status

Which of the following factors place a woman at risk for thrombosis. Select all that apply A. Obesity B. Physiological changes of pregnancy C. Metritis D. Cesarean birth

A. Obesity B. Physiological changes of pregnancy C. Metritis D. Cesarean birth

Signs and symptoms of postpartum depression include which of the following? Select all that apply A. Sleep and appetite disturbance B. Uncontrolled crying C. Delusions D. Feelings of guilt and/or worthlessness

A. Sleep and appetite disturbance B. Uncontrolled crying D. Feelings of guilt and/or worthlessness

When caring for a woman who speaks limited English, the appropriate method for translating information is: A. Using a trained medial interpreter B. Asking a student nurse who speaks the woman's primary language to translate C. Providing printed material for the woman to take home D. Asking for a family member to translate

A. Using a trained medical interpreter

Mrs. Fisher is 4 days post birth. She calls the clinic and tells the triage nurse that she has a temperature and does not feel well. What additional assessment findings does the triage nurse need to obtain to assist in her nursing assessment? Select all that apply A. When did she notice an increase in temperature and what is her temperature? B. is she experiencing pain and if so where? C. What is the amount of fluid intake within that past 24 hours D. What is the color and amount of her bleeding

A. When did she notice an increase in temperature and what is her temperature? B. is she experiencing pain and if so where? C. What is the amount of fluid intake within that past 24 hours D. What is the color and amount of her bleeding

Which of the following treatments is recommended for the infant experiencing drug withdrawal symptoms? A. morphine B. diluted formula C. frequent awakening D. well-lit room

A. morphine

Which of the following assessments would indicate instability in the client hospitalized for placenta previa? a. BP <90/60 mm/Hg, Pulse <60 BPM or >120 BPM b. FHR moderate variability without accelerations c. Dark brown vaginal discharge when voiding d. Oral temperature of 99.9F

ANS: A A decrease in BP accompanied by bradycardia or tachycardia is an indication of hypovolemic shock. FHR with moderate variability can be absent of accelerations during fetal sleep cycles or after maternal sedation. Bright red vaginal bleeding is an indication of current bleeding. Oral temperature may fluctuate based on the client's hydration status. It should be reassessed. Cause for concern is a temperature of 100.4 F or more.

The perinatal nurse is providing care to Marilyn, a 25-year-old G1 TPAL 0000 woman hospitalized with severe hypertension at 33 weeks' gestation. The nurse is preparing to administer the second dose of beta-methasone prescribed by the physician. Marilyn asks: "What is this injection for again?" The nurse's best response is: a. "This is to help your baby's lungs to mature." b. "This is to prepare your body to begin the labor process." c. "This is to help stabilize your blood pressure." d. "This is to help your baby grow and develop in preparation for birth."

ANS: A Antenatal glucocorticoids such as beta-methasone may be given (12 mg IM 24 hours apart) to promote fetal lung maturity if the gestational age is less than 34 weeks and childbirth can be delayed for 48 hours. Antenatal glucocorticoids such as beta-methasone may be given (12 mg IM 24 hours apart) to promote fetal lung maturity if the gestational age is less than 34 weeks and childbirth can be delayed for 48 hours. Antenatal glucocorticoids such as beta-methasone may be given (12 mg IM 24 hours apart) to promote fetal lung maturity if the gestational age is less than 34 weeks and childbirth can be delayed for 48 hours. Antenatal glucocorticoids such as beta-methasone may be given (12 mg IM 24 hours apart) to promote fetal lung maturity if the gestational age is less than 34 weeks and childbirth can be delayed for 48 hours.

For the patient with which of the following medical problems should the nurse question a physician's order for beta agonist tocolytics? a. Type 1 diabetes mellitus b. Cerebral palsy c. Myelomeningocele d. Positive group B streptococci culture

ANS: A Beta agonists often elevate serum glucose levels. The nurse should question the order. Beta agonists are not contraindicated for patients with cerebral palsy. Beta agonists are not contraindicated for patients with myelomeningocele. Beta agonists are not contraindicated for patients with group B streptococci.

Which of the following nursing diagnoses is of highest priority for a client with an ectopic pregnancy who has developed disseminated intravascular coagulation (DIC)? a. Risk for deficient fluid volume b. Risk for family process interrupted c. Risk for disturbed identity d. High risk for injury

ANS: A Correct. The client is at high risk for hypovolemia which is life threatening and takes precedence over any psychosocial or less pressing diagnoses. This is a psychosocial diagnosis and is not life threatening. This is a psychosocial diagnosis and is not life threatening. The client is at risk for injury; however, the diagnosis of deficient fluid volume is more descriptive and has clearly defined goals and interventions.

Identify the hallmark of placenta previa that differentiates it from abruptio placenta. a. Sudden onset of painless vaginal bleeding b. Board-like abdomen with severe pain c. Sudden onset of bright red vaginal bleeding d. Severe vaginal pain with bright red bleeding

ANS: A Correct. When the placenta attaches to the lower uterine segment near or over the cervical os, bleeding may occur without the onset of contractions or pain. The hallmark for abruptio placenta is pain and a board-like abdomen. Bright red bleeding could be related to abruptio placenta, placenta previa, or other complications of pregnancy. Pain is not a hallmark of placenta previa.

Metabolic changes during pregnancy __________ glucose tolerance. a. lower b. increase c. maintain d. alter

ANS: A Metabolic changes during pregnancy lower glucose tolerance.

A primiparous woman has been admitted at 35 weeks' gestation and diagnosed with HELLP syndrome. Which of the following laboratory changes is consistent with this diagnosis? a. Hematocrit dropped to 28%. b. Platelets increased to 300,000 cells/mm3. c. Red blood cells increased to 5.1 million cells/mm3. d. Sodium dropped to 132 mEq/dL.

ANS: A The nurse would expect to see a drop in the hematocrit: The H in HELLP stands for hemolysis. The nurse would expect to see low platelets. The nurse would expect to see hemolysis. The sodium is usually unaffected in HELLP syndrome.

Which of the following signs or symptoms would the nurse expect to see in a woman with concealed abruptio placentae? a. Increasing abdominal girth measurements b. Profuse vaginal bleeding c. Bradycardia with an aortic thrill d. Hypothermia with chills

ANS: A The nurse would expect to see increasing abdominal girth measurements.Profuse vaginal bleeding is rarely seen in placental abruption and is never seen when the abruption is concealed.With excessive blood loss, the nurse would expect to see tachycardia.The nurse would expect to see a stable temperature.

Your antepartal patient is 38 weeks' gestation, has a history of thrombosis, and has been on strict bed rest for the last 12 hours. She is now experiencing shortness of breath. What about the patient may be a contributing factor for her shortness of breath? a. Physiologic changes in pregnancy result in vasodilation, which increases the tendency to form blood clots. b. Physiologic changes in pregnancy result in vasoconstriction, which increases the tendency to form blood clots. c. Physiologic changes in pregnancy result in anemia, which increases the tendency to form blood clots. d. Physiologic changes in pregnancy result in decreased perfusion to the lungs, which increases the tendency to form blood clots.

ANS: A The patient's shortness of breath, bed rest, and history of thrombosis indicate possible pulmonary embolism. Her pregnant state also increases the potential for thrombosis resulting from increased levels of coagulation factors and decreased fibrinolysis, venous dilation, and obstruction of the venous system by the gravid uterus. Thromboembolitic diseases occurring most frequently in pregnancy include deep vein thrombosis and pulmonary embolism.

14. A 37-year-old gravid 8 para 7 woman was admitted to the postpartum unit at 2 hours postbirth. On admission to the unit, her fundus was U/U, midline, and firm, and her lochia was moderate rubra. An hour later, her fundus is midline and boggy, and the lochia is heavy with small clots. Based on this assessment data, the first nursing action is: a. Massage the fundus of the uterus. b. Assist the woman to the bathroom and reassess the fundus. c. Notify the physician or midwife. d. Start IV oxytocin therapy as per standing orders.

ANS: A a. Correct. Based on the assessment data that the uterus is midline and boggy, the woman is experiencing uterine atony. b. Assisting the woman to the bathroom would be a nursing action if the uterus was not midline. c. Oxytocin would be given and the primary health provider would be notified if the uterus did not respond to uterine massage. d. Oxytocin would be given and the primary health provider would be notified if the uterus did not respond to uterine massage.

1. A postpartum woman has been diagnosed with postpartum psychosis. Which of the following actions should the nurse perform? a. Supervise all infant care. b. Maintain client on strict bed rest. c. Restrict visitation to her partner. d. Carefully monitor toileting

ANS: A a. It is essential that a client diagnosed with postpartum (PP) psychosis not be left alone with her infant. b. There is no need for a client with PP psychosis to be on strict bed rest. c. Visitation is not usually restricted to the woman's partner. d. There is no need to monitor the client's toileting.

13. Which of the following is an indication for the administration of methylergonovine? a. Boggy uterus that does not respond to massage and oxytocin therapy b. Woman with a large hematoma c. Woman with a deep vein thrombosis d. Woman with severe postpartum depression

ANS: A a. Methylergonovine (methergine) is ordered for PPH due to uterine atony or subinvolution. It is used when massage and oxytocin therapy have failed to contract the uterus. b. Hematoma occurs when blood collects within the connective tissues of the vagina or perineal areas related to a vessel that ruptured and continues to bleed. Methylergonovine stimulates contraction of the smooth muscle of the uterus and would not have an effect on the vaginal or perineal areas. c. Heparin is usually prescribed for treatment of thrombosis. d. Methylergonovine is prescribed for treatment of uterine atony.

The perinatal nurse describes risk factors for placenta previa to the student nurse. Placenta previa risk factors include (select all that apply): a. Cocaine use b. Tobacco use c. Previous caesarean birth d. Previous use of medroxyprogesterone (Depo-Provera)

ANS: A, B, C Placenta previa may be associated with risk factors including smoking, cocaine use, a prior history of placenta previa, closely spaced pregnancies, African or Asian ethnicity, and maternal age greater than 35 years. Placenta previa may be associated with risk factors including smoking, cocaine use, a prior history of placenta previa, closely spaced pregnancies, African or Asian ethnicity, and maternal age greater than 35 years. Placenta previa may be associated with conditions that cause scarring of the uterus such as a prior cesarean section, multiparity, or increased maternal age. Previous use of medroxyprogesterone (Depo-Provera) is not a risk factor for placenta previa.

Marked hemodynamic changes in pregnancy can impact the pregnant woman with cardiac disease. Signs and symptoms of deteriorating cardiac status include (select all that apply): a. Orthopnea b. Nocturnal dyspnea c. Palpitations d. Irritation

ANS: A, B, C Signs and symptoms of deteriorating cardiac status with cardiac disease include orthopnea, nocturnal dyspnea, and palpitations, but do not include irritation.

The perinatal nurse provides a hospital tour for couples and families preparing for labor and birth in the future. Teaching is an important component of the tour. Information provided about preterm labor and birth prevention includes (select all that apply): a. Encouraging regular, ongoing prenatal care b. Reporting symptoms of urinary frequency and burning to the health-care provider c. Coming to the labor triage unit if back pain or cramping persist or become regular d. Lying on the right side, withholding fluids, and counting fetal movements if contractions occur every 5 minutes

ANS: A, B, C The nurse should encourage all pregnant women to obtain prenatal care and screen for vaginal and urogenital infections and treat appropriately, and remind pregnant women to call their provider repeatedly if symptoms of preterm labor occur. Educating all women of childbearing age about preterm labor is a crucial component of prevention. The nurse should encourage all pregnant women to obtain prenatal care and screen for vaginal and urogenital infections and treat appropriately, and remind pregnant women to call their provider repeatedly if symptoms of preterm labor occur. Educating all women of childbearing age about preterm labor is a crucial component of prevention. The nurse should encourage all pregnant women to obtain prenatal care and screen for vaginal and urogenital infections and treat appropriately, and remind pregnant women to call their provider if symptoms of preterm labor occur. Lying on the right side; drinking fluids, not withholding fluids; and counting fetal movements if contractions occur every 5 minutes are recommended if a woman thinks she is contracting.

Betamethasone is a steroid that is given to a pregnant woman with signs of preterm labor. The purpose of giving steroids is to (select all that apply): a. Stimulate the production of surfactant in the preterm infant b. Be given between 24 and 34 weeks' gestation c. Increase the severity of respiratory distress d. Accelerate fetal lung maturity

ANS: A, B, D Betamethasone is a steroid that is given to pregnant women with signs of preterm labor between 24 and 34 weeks' gestation. It stimulates the production of surfactant in the preterm infant and accelerates fetal lung maturity.

The perinatal nurse describes for the new nurse the various risks associated with prolonged premature preterm rupture of membranes. These risks include (select all that apply): a. Chorioamnionitis b. Abruptio placentae c. Operative birth d. Cord prolapse

ANS: A, B, D Even though maintaining the pregnancy to gain further fetal maturity can be beneficial, prolonged PPROM has been correlated with an increased risk of chorioamnionitis, placental abruption, and cord prolapse.

Kerry, a 30-year-old G3 TPAL 0110 woman presents to the labor unit triage with complaints of lower abdominal cramping and urinary frequency at 30 weeks' gestation. An appropriate nursing action would be to (select all that apply): a. Assess the fetal heart rate b. Obtain urine for culture and sensitivity c. Assess Kerry's blood pressure and pulse d. Palpate Kerry's abdomen for contractions

ANS: A, B, D Women experiencing preterm labor may complain of backache, pelvic aching, menstrual-like cramps, increased vaginal discharge, pelvic pressure, urinary frequency, and intestinal cramping with or without diarrhea. The patient's abdomen should be palpated to assess for contractions, and the fetus's heart rate should be monitored. Women experiencing preterm labor may complain of backache, pelvic aching, menstrual-like cramps, increased vaginal discharge, pelvic pressure, urinary frequency, and intestinal cramping with or without diarrhea. A urinalysis and urine culture and sensitivity (C & S) should be obtained on all patients who present with signs of preterm labor, and the nurse must remember that signs of UTI often mimic normal pregnancy complaints (i.e., urgency, frequency). The patient's abdomen should be palpated to assess for contractions, and the fetus's heart rate should be monitored. Assessment of blood pressure and pulse is not an important nursing action in this scenario. Women experiencing preterm labor may complain of backache, pelvic aching, menstrual-like cramps, increased vaginal discharge, pelvic pressure, urinary frequency, and intestinal cramping with or without diarrhea. The patient's abdomen should be palpated to assess for contractions and the fetus's heart rate should be monitored.

81. Verbalizing someone else's wishes if he or she is unable to do so

ANS: Advocacy Refer To: Chapter 2 KEY: Integrated Process: Teaching and Learning | Cognitive Level: Knowledge | Content Area: Maternity | Client Need: Health Promotion and Maintenance | Difficulty Level: Easy

82. Absence of menses

ANS: Amenorrhea Refer To: Glossary KEY: Integrated Process: Teaching and Learning | Cognitive Level: Knowledge | Content Area: Maternity | Client Need: Health Promotion and Maintenance | Difficulty Level: Easy

While educating the client with class II cardiac disease, at 28 weeks' gestation, the nurse instructs the client to notify the physician if she experiences which of the following conditions? a. Emotional stress at work b. Increased dyspnea while resting c. Mild pedal and ankle edema d. Weight gain of 1 pound in 1 week

ANS: B Emotional stress increases cardiac workload; however, without symptoms of cardiac decompensation, this is not immediately concerning. Increasing dyspnea, at rest, can be a sign of cardiac decompensation leading to increased congestive heart failure. Mild edema during the third trimester is normal. However, increasing edema and pitting edema should be reported as they can be a sign of increasing CHF. A weight gain of 1 pound per week is expected during the third trimester.

A labor nurse is caring for a patient, 39 weeks' gestation, who has been diagnosed with placenta previa. Which of the following physician orders should the nurse question? a. Type and cross-match her blood. b. Insert an internal fetal monitor electrode. c. Administer an oral stool softener. d. Assess her complete blood count.

ANS: B It would be appropriate to type and cross-match the patient for a blood transfusion. This action is inappropriate. When a patient has a placenta previa, nothing should be inserted into the vagina. To prevent constipation, it is appropriate for a patient to take a stool softener. It is appropriate to monitor the patient for signs of anemia.

A 34-weeks' gestation multigravida, G3 P1 is admitted to the labor suite. She is contracting every 7 minutes and 40 seconds. The woman has several medical problems. Which of the following of her comorbidities is most consistent with the clinical picture? a. Kyphosis b. Urinary tract infection c. Congestive heart failure d. Cerebral palsy

ANS: B Kyphosis is unrelated to preterm labor. Urinary tract infections often precipitate preterm labor. It is unlikely that the congestive heart failure precipitated the preterm labor. Cerebral palsy is unrelated to preterm labor.

During pregnancy, poorly controlled asthma can place the fetus at risk for: a. Hyperglycemia b. IUGR c. Hypoglycemia d. Macrosomia

ANS: B Maternal asthma does not place the fetus at risk for hyperglycemia. Compromised pulmonary function can lead to decompensation and hypoxia that decrease oxygen flow to the fetus and can cause intrauterine growth restriction (IUGR). Asthma does not directly affect glycemic control. A fetus experiencing hypoxia would be small for gestational age, not large for gestational age.

A woman at 32 weeks' gestation is diagnosed with severe preeclampsia with HELLP syndrome. The nurse will identify which of the following as a positive patient care outcome? a. Rise in serum creatinine b. Drop in serum protein c. Resolution of thrombocytopenia d. Resolution of polycythemia

ANS: C A rise in serum creatinine indicates that the kidneys are not effectively excreting creatinine. It is a negative outcome. A drop in serum protein indicates that the kidneys are allowing protein to be excreted. This is a negative outcome. Resolution of thrombocytopenia is a positive sign. It indicates that the platelet count is returning to normal. Polycythemia is not related to HELLP syndrome. Rather one sees a drop in red cell and platelet counts with HELLP. A positive sign, therefore, would be a rise in the RBC count.

A woman who is 36 weeks pregnant presents to the labor and delivery unit with a history of congestive heart disease. Which of the following findings should the nurse report to the primary health-care practitioner? a. Presence of chloasma b. Presence of severe heartburn c. 10-pound weight gain in a month d. Patellar reflexes +1

ANS: C Chloasma is a normal pregnancy finding. Heartburn is an expected finding during the third trimester. The weight gain may be due to fluid retention. Fluid retention may occur in patients with pregnancy-induced hypertension and in patients with congestive heart failure. The physician should be notified. Although slightly hyporeflexic, patellar reflexes of +1 are within normal limits.

You are caring for a patient who was admitted to labor and delivery at 32 weeks' gestation and diagnosed with preterm labor. She is currently on magnesium sulfate, 2 gm per hour. Upon your initial assessment you note that she has a respiratory rate of 8 with absent deep tendon reflexes. What will be your first nursing intervention? a. Elevate head of the bed b. Notify the MD c. Discontinue magnesium sulfate d. Draw a serum magnesium level

ANS: C Initial nursing intervention needs to be discontinuing magnesium sulfate because the patient is exhibiting signs of magnesium toxicity with absent deep tendon reflexes and decreased respiratory rate

A pregnant client with a history of multiple sexual partners is at highest risk for which of the following complications: a. Premature rupture of membranes b. Gestational diabetes c. Ectopic pregnancy d. Pregnancy-induced hypertension

ANS: C Multiple partners do not increase a woman's risk of premature rupture of membranes. Genetics and client diet and weight are contributing factors to gestational diabetes. Correct. A history of multiple sexual partners places the client at a higher risk of having contracted a sexually transmitted disease that could have ascended the uterus to the fallopian tubes and caused fallopian tube blockage, placing the client at high risk for an ectopic pregnancy. Multiple sexual partners are not a risk factor for pregnancy-induced hypertension.

A woman who has had no prenatal care was assessed and found to have hydramnios on admission to the labor unit and has since delivered a baby weighing 4500 grams. Which of the following complications of pregnancy likely contributed to these findings? a. Pyelonephritis b. Pregnancy-induced hypertension c. Gestational diabetes d. Abruptio placentae

ANS: C Pyelonephritis does not lead to the development of hydramnios or macrosomia. Pregnancy-induced hypertension does not lead to the development of hydramnios or macrosomia. Untreated gestational diabetics often have hydramnios and often deliver macrosomic babies. Abruptio placentae does not lead to the development of hydramnios or macrosomia.

A patient is receiving magnesium sulfate for severe preeclampsia. The nurse must notify the attending physician immediately of which of the following findings? a. Patellar and biceps reflexes of +4 b. Urinary output of 50 mL/hr c. Respiratory rate of 10 rpm d. Serum magnesium level of 5 mg/dL

ANS: C The magnesium sulfate has been ordered because the patient has severe pregnancy-induced hypertension. Patellar and biceps reflexes of +4 are symptoms of the disease. The urinary output must be above 25 mL/hr. The drop in respiratory rate may indicate that the patient is suffering from magnesium toxicity. The nurse should report the finding to the physician. The therapeutic range of magnesium is 4 to 7 mg/dL.

A woman who is admitted to labor and delivery at 30 weeks' gestation, is 1 cm dilated, and is contracting q 5 minutes. She is receiving magnesium sulfate IV piggyback. Which of the following maternal vital signs is most important for the nurse to assess each hour? a. Temperature b. Pulse c. Respiratory rate d. Blood pressure

ANS: C The temperature should be monitored, but it is not the most important vital sign. The pulse rate should be monitored, but it is not the most important vital sign. The respiratory rate is the most important vital sign. Respiratory depression is a sign of magnesium toxicity. The blood pressure should be monitored, but it is not the most important vital sign.

A client on 2 gm/hr of magnesium sulfate has decreased deep tendon reflexes. Identify the priority nursing assessment to ensure client safety. a. Assess uterine contractions continuously b. Assess fetal heart rate continuously c. Assess urinary output d. Assess respiratory rate

ANS: D Monitoring contractions does not indicate magnesium toxicity. Magnesium sulfate will decrease fetal variability and not provide an accurate assessment of magnesium toxicity. Urinary output does not correlate to decreased deep tendon reflexes. Correct. Respiratory effort and deep tendon reflexes (DTRs) are involuntary, and a decrease in DTRs could indicate the risk of magnesium sulfate toxicity and the risk for decreased respiratory effort.

The nurse is caring for two laboring women. Which of the patients should be monitored most carefully for signs of placental abruption? a. The patient with placenta previa b. The patient whose vagina is colonized with group B streptococci c. The patient who is hepatitis B surface antigen positive d. The patient with eclampsia

ANS: D Patients with placenta previa are not especially high risk for placental abruption. Patients colonized with group B streptococci are not especially high risk for placental abruption. Patients who are hepatitis B surface antigen positive are not especially high risk for placental abruption. Patients with eclampsia are high risk for placental abruption.

The perinatal nurse knows that the term to describe a woman at 26 weeks' gestation with a history of elevated blood pressure who presents with a urine showing 2+ protein (by dipstick) is: a. Preeclampsia b. Chronic hypertension c. Gestational hypertension d. Chronic hypertension with superimposed preeclampsia

ANS: D Preeclampsia is a multisystem, vasopressive disease process that targets the cardiovascular, hematologic, hepatic, and renal and central nervous systems. Chronic hypertension is hypertension that is present and observable prior to pregnancy or hypertension that is diagnosed before the 20th week of gestation. Gestational hypertension is a nonspecific term used to describe the woman who has a blood pressure elevation detected for the first time during pregnancy, without proteinuria. The following criteria are necessary to establish a diagnosis of superimposed preeclampsia: hypertension and no proteinuria early in pregnancy (prior to 20 weeks' gestation) and new-onset proteinuria, a sudden increase in protein—urinary excretion of 0.3 g protein or more in a 24-hour specimen, or two dipstick test results of 2+ (100 mg/dL), with the values recorded at least 4 hours apart, with no evidence of urinary tract infection; a sudden increase in blood pressure in a woman whose blood pressure has been well controlled; thrombocytopenia (platelet count lower than 100,000/mmC); and an increase in the liver enzymes alanine transaminase (ALT) or aspartate transaminase (AST) to abnormal levels.

The perinatal nurse is assessing a woman in triage who is 34 + 3 weeks' gestation in her first pregnancy. She is worried about having her baby "too soon," and she is experiencing uterine contractions every 10 to 15 minutes. The fetal heart rate is 136 beats per minute. A vaginal examination performed by the health-care provider reveals that the cervix is closed, long, and posterior. The most likely diagnosis would be: a. Preterm labor b. Term labor c. Back labor d. Braxton-Hicks contractions

ANS: D Preterm labor (PTL) is defined as regular uterine contractions and cervical dilation before the end of the 36th week of gestation. Many patients present with preterm contractions, but only those who demonstrate changes in the cervix are diagnosed with preterm labor. Term labor occurs after 37 weeks' gestation. There is no indication in this scenario that this is back labor. Braxton-Hicks contractions are regular contractions occurring after the third month of pregnancy. They may be mistaken for regular labor, but unlike true labor, the contractions do not grow consistently longer, stronger, and closer together, and the cervix is not dilated. Some patients present with preterm contractions, but only those who demonstrate changes in the cervix are diagnosed with preterm labor.

The nurse is caring for a woman at 28 weeks' gestation with a history of preterm delivery. Which of the following laboratory data should the nurse carefully assess in relation to this diagnosis? a. Human relaxin levels b. Amniotic fluid levels c. Alpha-fetoprotein levels d. Fetal fibronectin levels

ANS: D Relaxin levels are rarely assessed. In addition, they are unrelated to the incidence of preterm labor. Amniotic fluid levels are not directly related to the incidence of preterm labor. Alpha-fetoprotein levels are not related to the incidence of preterm labor. A rise in the fetal fibronectin levels in cervical secretions has been associated with preterm labor.

According to agency policy, the perinatal nurse provides the following intrapartal nursing care for the patient with preeclampsia: a. Take the patient's blood pressure every 6 hours b. Encourage the patient to rest on her back c. Notify the physician of a urine output greater than 30 mL/hr d. Administer magnesium sulfate according to agency policy

ANS: D The nurse is the manager of care for the woman with preeclampsia during the intrapartal period. Careful assessments are critical. The blood pressure is taken every 1 hour or more frequently according to physician orders or institutional protocol. The nurse is the manager of care for the woman with preeclampsia during the intrapartal period. Careful assessments are critical. The patient should be encouraged to assume a side-lying position to enhance uterine perfusion. The nurse is the manager of care for the woman with preeclampsia during the intrapartal period. Careful assessments are critical. A urine output less than 30 mL/hr is indicative of oliguria and the physician must be notified. The nurse is the manager of care for the woman with preeclampsia during the intrapartal period. Careful assessments are critical. The nurse administers medications as ordered and should adhere to hospital protocol for a magnesium sulfate infusion.

A woman in labor and delivery is being given subcutaneous terbutaline for preterm labor. Which of the following common medication effects would the nurse expect to see in the mother? a. Serum potassium level increases b. Diarrhea c. Urticaria d. Complaints of nervousness

ANS: D The nurse would not expect to see a rise in the mother's serum potassium levels. The beta agonists are not associated with diarrhea. The beta agonists are not associated with urticaria. Complaints of nervousness are commonly made by women receiving subcutaneous beta agonists.

A type 1 diabetic patient has repeatedly experienced elevated serum glucose levels throughout her pregnancy. Which of the following complications of pregnancy would the nurse expect to see? a. Postpartum hemorrhage b. Neonatal hyperglycemia c. Postpartum oliguria d. Neonatal macrosomia

ANS: D The patient is not especially high risk for a postpartum hemorrhage. The nurse would expect to see neonatal hypoglycemia, not hyperglycemia. The nurse would expect to see postpartum polyuria. The nurse would expect to see neonatal macrosomia.

The nurse working in a prenatal clinic is providing care to three primigravida patients. Which of the patient findings would the nurse highlight for the physician? a. 15 weeks, denies feeling fetal movement b. 20 weeks, fundal height at the umbilicus c. 25 weeks, complains of excess salivation d. 30 weeks, states that her vision is blurry

ANS: D This finding is normal. Quickening is usually felt between 16 and 20 weeks' gestation. This finding is normal. The fundal height at 20 weeks' gestation is usually at the level of the umbilicus. Excess salivation is a normal, albeit annoying, finding. Blurred vision is a sign of pregnancy-induced hypertension (PIH). This finding should be reported to the woman's health-care practitioner.

Which of the following statements is most appropriate for the nurse to say to a patient with a complete placenta previa? a. "During the second stage of labor you will need to bear down." b. "You should ambulate in the halls at least twice each day." c. "The doctor will likely induce your labor with oxytocin." d. "Please promptly report if you experience any bleeding or feel any back discomfort."

ANS: D This response is inappropriate. This patient will be delivered by cesarean section. This response is inappropriate. Patients with placenta previa are usually on bed rest. This response is inappropriate. This patient will be delivered by cesarean section. Labor often begins with back pain. Labor is contraindicated for a patient with complete placenta previa.

3. A client is 1 hour postpartum from a vacuum delivery over a midline episiotomy of a 4500-gram neonate. Which of the following nursing diagnoses is appropriate for this mother? a. Risk for altered parenting b. Risk for imbalanced nutrition: less than body requirements c. Risk for ineffective individual coping d. Risk for fluid volume deficit

ANS: D a. Although the baby is macrosomic, there is no evidence that this mother is high risk for altered parenting. b. This woman's baby is macrosomic—there is no indication that this woman is consuming a diet that is less than body requirements. c. There is no evidence that this mother is high risk for altered coping. d. This client is high risk for fluid volume deficit. Women who deliver macrosomic babies are high risk for uterine atony, which can lead to heavy flow of lochia.

67. The perinatal nurse explains to the new nurse that ptyalism is a condition more acute than the normal nausea and vomiting of pregnancy and is often associated with dehydration, hypokalemia, and weight loss.

ANS: False Hyperemesis gravidarum is a pregnancy-related condition characterized by persistent, continuous, severe nausea and vomiting, often accompanied by dry retching. Hyperemesis gravidarum results in weight loss and fluid and electrolyte imbalance. Ptyalism is an excessive production of saliva. KEY: Integrated Process: Teaching and Learning | Cognitive Level: Application | Content Area: Maternity | Client Need: Safe and Effective Care Environment | Difficulty Level: Moderate

The perinatal nurse explains to the student nurse that the most frequent fetal risk associated with the use of forceps is cord compression.

ANS: False The most frequent fetal risk associated with the use of forceps is superficial scalp or facial marks that will resolve quickly.

__________ is contraindicated with shoulder dystocia.

ANS: Fundal pressureFundal pressure is contraindicated with shoulder dystocia because it may further impact the shoulder and increases risk of fetal injury.

83. Curvature of the spine

ANS: Lordosis Refer To: Glossary KEY: Integrated Process: Teaching and Learning | Cognitive Level: Knowledge | Content Area: Peds/Maternity | Client Need: Health Promotion and Maintenance | Difficulty Level: Easy

58. Anterior convexity of the lumbar spine

ANS: Lumbar lordosis Refer To: Glossary KEY: Integrated Process: Teaching and Learning | Cognitive Level: Knowledge | Content Area: Maternity | Client Need: Health Promotion and Maintenance | Difficulty Level: Easy

61. Severe itching due to stasis of bile in the liver

ANS: Pruritis gravidarum Refer To: Glossary KEY: Integrated Process: Teaching and Learning | Cognitive Level: Knowledge | Content Area: Peds/Maternity | Client Need: Health Promotion and Maintenance | Difficulty Level: Easy

59. Increased saliva production

ANS: Ptyalism Refer To: Glossary KEY: Integrated Process: Teaching and Learning | Cognitive Level: Knowledge | Content Area: Maternity | Client Need: Health Promotion and Maintenance | Difficulty Level: Easy

60. Reflux of the stomach contents into the esophagus

ANS: Pyrosis Refer To: Glossary KEY: Integrated Process: Teaching and Learning | Cognitive Level: Knowledge | Content Area: Maternity | Client Need: Health Promotion and Maintenance | Difficulty Level: Easy

84. Stretch marks

ANS: Striae gravidarum Refer To: Glossary KEY: Integrated Process: Teaching and Learning | Cognitive Level: Knowledge | Content Area: Maternity | Client Need: Health Promotion and Maintenance | Difficulty Level: Easy

A hematoma is the collection of blood beneath the intact skin layer following an injury to a blood vessel. T/F

ANS: True A hematoma is a localized collection of blood in connective or soft tissue under the skin that follows injury of or laceration to a blood vessel without injury to the overlying tissue. At the time of injury, pressure necrosis and inadequate hemostasis occur.

The perinatal nurse includes the following when explaining the physiology of artificial rupture of membranes to the student nurse: rupture of membranes causes a release of arachidonic acid, which converts to prostaglandins, substances known to stimulate oxytocin in the pregnant uterus.

ANS: True At certain points in the labor, an amniotomy, or artificial rupture of the membranes, may be successful in increasing uterine contractility.

65. The clinic nurse knows that every time a woman of childbearing age comes in to the office for a health maintenance visit, she should be counseled about the benefits of daily folic acid supplementation.

ANS: True Because of the strong connection between folic acid deficiency and the subsequent development of neural tube defects, all women of childbearing age should take a folic acid supplement of at least 400 mcg/day. KEY: Integrated Process: Nursing Process: Clinical Problem Solving | Cognitive Level: Application | Content Area: Maternity | Client Need: Safe and Effective Care Environment | Difficulty Level: Moderate

64. Cecilia, a pregnant woman at 30 weeks' gestation, has her vital signs assessed during a routine prenatal visit. Cecilia's blood pressure has remained at 110/70 for the last few visits, and her pulse rate has increased from 70 to 80 beats per minute. These findings would be considered normal at this time in pregnancy.

ANS: True During the first trimester, blood pressure normally remains the same as prepregnancy levels but then gradually decreases up to around 20 weeks' of gestation. After 20 weeks, the vascular volume expands and the blood pressure increases to reach prepregnant levels by term. KEY: Integrated Process: Nursing Process: Clinical Problem Solving | Cognitive Level: Knowledge | Content Area: Maternity | Client Need: Safe and Effective Care Environment | Difficulty Level: Easy

63. The clinic nurse speaks with the student nurse prior to the physical examination of a pregnant woman who is 32 weeks' gestation. The clinic nurse explains that the heart sounds heard in pregnancy are usually S1 and S3 with a possible murmur related to increased cardiac output.

ANS: True Exaggerated first and third heart sounds and systolic murmurs are common findings during pregnancy. The murmurs are usually asymptomatic and require no treatment. KEY: Integrated Process: Teaching and Learning | Cognitive Level: Application | Content Area: Maternity | Client Need: Safe and Effective Care Environment | Difficulty Level: Moderate

The perinatal nurse describes asynclitism to students as a presentation that occurs when the fetal head is turned toward the maternal sacrum or symphysis at an oblique angle.

ANS: True Face and brow presentations are examples of asynclitism (the fetal head is presenting at a different angle than expected). Face and brow presentations hyperextend the neck and increase the overall circumference of the presenting part. These presentations are uncommon and are usually associated with fetal anomalies.

Metritis is an infection that usually starts at the placental site. T/F

ANS: True Metritis is an infection of the endometrium that usually starts at the placental site and spreads to encompass the entire endometrium.

66. The perinatal nurse recommends strengthening exercises during pregnancy, as this can improve posture and increase energy levels.

ANS: True Muscle strengthening benefits the woman as she copes with the physical changes of pregnancy, which include weight gain and postural changes. Muscle strengthening exercises also help to decrease the risk of ligament and joint injury. KEY: Integrated Process: Teaching and Learning | Cognitive Level: Application | Content Area: Maternity | Client Need: Safe and Effective Care Environment | Difficulty Level: Moderate

Abruptio placenta is a risk factor for amniotic fluid embolism. T/F

ANS: True Risk factors for amniotic fluid embolism include induction of labor, maternal age over 35, operative delivery, placenta previa, abruptio placenta, polyhydramnios, eclampsia, and cervical or uterine lacerations

6. The perinatal nurse recognizes that a risk factor for postpartum depression is: a. Inadequate social support b. Age >35 years c. Gestational hypertension d. Regular schedule of prenatal care

ANS: a Recognized risk factors for postpartum depression include an undesired or unplanned pregnancy, a history of depression, recent major life changes such as the death of a family member, moving to a new community, lack of family or social support, financial stress, marital discord, adolescent age, and homelessness.

15. The perinatal nurse is assisting the student nurse with completion of documentation. The laboring woman has just given birth to a 2700 gram infant at 36 weeks' gestation. The most appropriate term for this is: a. Preterm birth b. Term birth c. Small for gestational age infant d. Large for gestational age infant

ANS: a a. A preterm infant is an infant with gestational age of fewer than 36 completed weeks. b. Term births are infants born between 37 and 40 weeks. c. SAG infants at 36 weeks weigh less than 2000 grams. d. LAG infants at 36 weeks weigh over 3400 grams.

10. A 42-week gestation neonate is admitted to the NICU (neonatal intensive care unit). This neonate is at risk for which complication? a. Meconium aspiration syndrome b. Failure to thrive c. Necrotizing enterocolitis d. Intraventricular hemorrhage

ANS: a a. Although there is nothing in the scenario that states that the amniotic fluid is green tinged, post-term babies are high risk for meconium aspiration syndrome. b. Post-term babies often gain weight very quickly. c. Preterm, not post-term, babies are high risk for necrotizing enterocolitis. d. Preterm, not post-term, babies are high risk for intraventricular hemorrhages.

A pregnant woman who has a history of cesarean births is requesting to have a vaginal birth after cesarean (VBAC). In which of the following situations should the nurse advise the patient that her request may be declined? a. Transverse fetal lie b. Flexed fetal attitude c. Previous low flap uterine incision d. Positive vaginal candidiasis

ANS: a A baby in the transverse lie is lying sideways in the uterus. This lie is incompatible physiologically with a vaginal delivery.

Four women are close to delivery on the labor and delivery unit. The nurse knows to be vigilant to the signs of neonatal respiratory distress in which delivery? a. 42-week-gestation pregnancy complicated by intrauterine growth restriction b. 41-week-gestation pregnancy with biophysical profile score of 10 that morning c. 40-week-gestation pregnancy with estimated fetal weight of 3200 grams d. 39-week-gestation pregnancy complicated by maternal cholecystitis

ANS: a A post-term baby with intrauterine growth restriction (IUGR) is high risk for meconium aspiration syndrome, cold stress syndrome, hypoglycemia, and acidosis. In each case, the baby may exhibit signs of respiratory distress.

Augmentation of labor: a. Is part of the active management of labor instituted when the labor process is unsatisfactory and uterine contractions are inadequate b. Relies on more invasive methods when oxytocin and amniotomy have failed c. Is elective induction of labor d. Is an operative vaginal delivery that uses vacuum cups

ANS: a Augmentation stimulates uterine contractions after labor has started but not progressed appropriately.

The nurse is interviewing a pregnant client who states she plans to drink chamomile tea to ensure an effective labor. The nurse knows that this is an example of: a. Cultural prescription b. Cultural taboo c. Cultural restriction d. Cultural demonstration

ANS: a Correct. Cultural prescription is an expected behavior of the pregnant woman during the childbearing period.

29. A nurse who is discussing serving sizes of foods with a new prenatal patient would state that which of the following is equal to 1 (one) serving from the dairy food group? a.. 1 cup low-fat milk b. ½ cup vanilla yogurt c. ½ cup cottage cheese d.. 1 ounce cream cheese

ANS: a Feedback a. 1 cup of any milk (e.g., whole milk, skim milk, buttermilk, chocolate milk) is equal to 1 serving size from the dairy group. b. 1 cup of yogurt is equal to 1 serving size from the dairy group. c. 1 ½ cup of cottage cheese is equal to 1 serving size from the dairy group. d. Cream cheese is not included in the dairy group. It is a fat product. KEY: Integrated Process: Nursing Process: Implementation; Teaching and Learning | Cognitive Level: Comprehension | Content Area: Antepartum Care; Basic Care and Comfort: Nutrition | Client Need: Health Promotion and Maintenance; Physiological Integrity: Basic Care and Comfort | Difficulty Level: Easy

21. A patient at 37 weeks' gestation is being seen in the prenatal clinic. Where would the nurse expect the fundal height to be palpated? a. At the xiphoid process b. At a point between the umbilicus and the xiphoid c. At the umbilicus d. At a level directly above the symphysis pubis

ANS: a Feedback a. At 36 weeks' gestation, the fundus should be felt at the xiphoid process. b. At 36 weeks' gestation, the fundus should be felt at the xiphoid process. c. At 20 weeks' gestation, the fundus should be felt at the umbilicus. d. At 12 weeks' gestation, the fundus should be felt directly above the symphysis pubis. KEY: Integrated Process: Nursing Process: Assessment | Cognitive Level: Comprehension | Content Area: Antepartum Care | Client Need: Health Promotion and Maintenance | Difficulty Level: Easy

6. A woman presents to the prenatal clinic at 30 weeks' gestation reporting dysuria, frequency, and urgency with urination. Appropriate nursing actions include: a. Obtain clean-catch urine to assess for a possible urinary tract infection. b. Reassure the woman that the signs are normal urinary changes in the third trimester. c. Teach the woman to decrease fluid intake to manage these symptoms. d. Perform a Leopold's maneuver to assess fetal position and station.

ANS: a Feedback a. Correct. Dysuria, frequency, and urgency with urination are signs and symptoms of a urinary tract infection, necessitating further assessment and testing. b. These are abnormal urinary symptoms in the third trimester. c. Pregnant women need to increase their fluid intake during pregnancy, and dysuria and urgency are abnormal. d. Assessment of fetal position and station is not an appropriate response to reported signs and symptoms of a urinary tract infection. KEY: Integrated Process: Clinical Problem Solving | Cognitive Level: Application | Content Area: Maternity | Client Need: Physiological Adaptation | Difficulty Level: Moderate

32. The clinic nurse includes screening for domestic violence in the first prenatal visit for all patients. An appropriate question would be: a. This is something that we ask everyone. Do you feel safe in your current living environment and relationships? b. This is something we ask everyone. Do you have any abuse in your life right now? c. Is your partner threatening or harming you in any way right now? d. I need to ask you, do you feel safe from abuse right now?

ANS: a Feedback a. Intimate partner violence is a difficult subject to discuss, and the nurse may fear insulting or psychologically hurting the patient more. A nonthreatening approach is to ask patients directly whether they feel safe going home and whether they have been hurt physically, emotionally, or sexually by a past or present partner. b. Intimate partner violence is a difficult subject to discuss, and the nurse may fear insulting or psychologically hurting the patient more. A nonthreatening approach is to ask patients directly whether they feel safe going home rather than asking if they have any abuse, as women may define abuse differently than care providers. c. Intimate partner violence is a difficult subject to discuss, and the nurse may fear insulting or psychologically hurting the patient more. A nonthreatening approach is to ask patients directly whether they feel safe going home and whether they have been hurt physically, emotionally, or sexually by a past or present partner. d. Intimate partner violence is a difficult subject to discuss, and the nurse may fear insulting or psychologically hurting the patient more. A nonthreatening approach is to ask patients directly whether they feel safe going home rather than asking if they have any abuse, as women may define abuse differently than care providers. KEY: Integrated Process: Nursing Process: Clinical Problem Solving | Cognitive Level: Application | Content Area: Maternity | Client Need: Safe and Effective Care Environment | Difficulty Level: Moderate

12. The clinic nurse uses Leopold maneuvers to determine the fetal lie, presentation, and position. The nurse's hands are placed on the maternal abdomen to gently palpate the fundal region of the uterus. This action is best described as the: a. First maneuver b. Second maneuver c. Third maneuver d. Fourth maneuver

ANS: a Feedback a. Leopold maneuvers are a four-part clinical assessment method used to determine the lie, presentation, and position of the fetus. The first maneuver determines which fetal body part (e.g., head or buttocks) occupies the uterine fundus. The examiner faces the patient's head and places the hands on the abdomen, using the palmar surface of the hands to gently palpate the fundal region of the uterus. The buttocks feel soft, broad, and poorly defined and move with the trunk. The fetal head feels firm and round and moves independently of the trunk. b. Leopold maneuvers are a four-part clinical assessment method used to determine the lie, presentation, and position of the fetus. The first maneuver is described in this scenario. c. Leopold maneuvers are a four-part clinical assessment method used to determine the lie, presentation, and position of the fetus. The first maneuver is described in this scenario. d. Leopold maneuvers are a four-part clinical assessment method used to determine the lie, presentation, and position of the fetus. The first maneuver is described in this scenario. KEY: Integrated Process: Nursing Process: Clinical Problem Solving | Cognitive Level: Application | Content Area: Maternity | Client Need: Safe and Effective Care Environment | Difficulty Level: Moderate

14. Lina is an 18-year-old woman at 20 weeks' gestation. This is her first pregnancy. Lina is complaining of fatigue and listlessness. Her vital signs are within a normal range: BP = 118/60, pulse = 70, and respiratory rate 16 breaths per minute. Lina's fundal height is at the umbilicus, and she states that she is beginning to feel fetal movements. Her weight gain is 25 pounds over the prepregnant weight (110 lb), and her height is 5 feet 4 inches. The perinatal nurse's best approach to care at this visit is to: a. Ask Lina to keep a 3-day food diary to bring in to her next visit in 1 week. b. Explain to Lina that weight gain is not a concern in pregnancy, and she should not worry. c. Teach Lina about the expected normal weight gain during pregnancy (approximately 20 pounds by 20 weeks' gestation). d. Explain to Lina the possible concerns related to excessive weight gain in pregnancy, including the risk of gestational diabetes.

ANS: a Feedback a. Nutrition and weight management play an essential role in the development of a healthy pregnancy. Not only does the patient need to have an understanding of the essential nutritional elements, she must also be able to assess and modify her diet for the developing fetus and her own nutritional maintenance. To facilitate this process, it is the nurse's responsibility to provide education and counseling concerning dietary intake, weight management, and potentially harmful nutritional practices. To facilitate this process, it is the nurse's responsibility to gather more information on the woman's dietary practices through a food diary. b. Nutrition and weight management play an essential role in the development of a healthy pregnancy. To facilitate this process, it is the nurse's responsibility to provide education and counseling concerning dietary intake, weight management, and potentially harmful nutritional practices. c. Nutrition and weight management play an essential role in the development of a healthy pregnancy. Not only does the patient need to have an understanding of the essential nutritional elements, she must also be able to assess and modify her diet for the developing fetus and her own nutritional maintenance. To facilitate this process, it is the nurse's responsibility to provide education and counseling concerning dietary intake, weight management, and potentially harmful nutritional practices, not just inform the patient of expected normal weight gain. d. Nutrition and weight management play an essential role in the development of a healthy pregnancy. Not only does the patient need to have an understanding of the essential nutritional elements, she must also be able to assess and modify her diet for the developing fetus and her own nutritional maintenance. To facilitate this process, it is the nurse's responsibility to provide education and counseling concerning dietary intake, weight management, and potentially harmful nutritional practices. KEY: Integrated Process: Nursing Process: Clinical Problem Solving | Cognitive Level: Application | Content Area: Peds/Maternity | Client Need: Safe and Effective Care Environment | Difficulty Level: Moderate

18. A nurse is reviewing diet with a pregnant woman in her second trimester. Which of the following foods should the nurse advise the patient to avoid consuming during her pregnancy? a. Brie cheese b. Bartlett pears c. Sweet potatoes d. Grilled lamb

ANS: a Feedback a. Soft cheese may harbor Listeria. The patient should avoid consuming uncooked soft cheese. b. A pear is an excellent food for a pregnant woman to consume. c. Sweet potatoes are an excellent food for a pregnant woman to consume. d. Grilled lamb is an excellent food for a pregnant woman to consume, although it should be well cooked. KEY: Integrated Process: Nursing Process: Implementation; Teaching and Learning | Cognitive Level: Application | Content Area: Antepartum Care; Reduction of Risk Potential: Potential for Alterations in Body Systems | Client Need: Health Promotion and Maintenance; Physiological Integrity: Reduction of Risk Potential | Difficulty Level: Moderate

27. A nurse working in a prenatal clinic is caring for a woman who asks advice on foods that are high in vitamin C because "I hate oranges." The nurse states that 1 cup of which of the following raw foods will meet the patient's daily vitamin C needs? a. Strawberries b. Asparagus c. Iceberg lettuce d. Cucumber

ANS: a Feedback a. Strawberries are an excellent source of vitamin C. b. Although asparagus has some vitamin C, it is not an excellent source. c. Iceberg lettuce is a poor source of vitamin C. d. Cucumber is a poor source of vitamin C. KEY: Integrated Process: Nursing Process: Implementation; Teaching and Learning | Cognitive Level: Knowledge | Content Area: Health and Wellness | Client Need: Health Promotion and Maintenance | Difficulty Level: Easy

20. A gravida, G4 P1203, fetal heart rate 150s, is 14 weeks pregnant, fundal height 1 cm above the symphysis. She denies experiencing quickening. Which of the following nursing conclusions made by the nurse is correct? a. The woman is experiencing a normal pregnancy. b. The woman may be having difficulty accepting this pregnancy. c. The woman must see a nutritionist as soon as possible. d. The woman will likely miscarry the conceptus.

ANS: a Feedback a. The patient is experiencing a normal pregnancy. b. Quickening is not felt until 16 to 20 weeks' gestation. c. There is no apparent need for a nutritionist to see this patient. d. There is no indication in the scenario that this patient is at high risk for a miscarriage. KEY: Integrated Process: Nursing Process: Analysis | Cognitive Level: Application | Content Area: Antepartum Care | Client Need: Health Promotion and Maintenance | Difficulty Level: Moderate

9. A 26-year-old woman at 29 weeks' gestation experienced epigastric pain following the consumption of a large meal of fried fish and onion rings. The pain resolved a few hours later. The most likely diagnosis for this symptom is: a. Cholelithiasis b. Influenza c. Urinary tract infection d. Indigestion

ANS: a Feedback a. The progesterone-induced prolonged emptying time of bile from the gallbladder, combined with elevated blood cholesterol levels, may predispose the pregnant woman to gallstone formation (cholelithiasis). Pain in the epigastric region following ingestion of a high-fat meal constitutes the major symptom of these conditions. The pain is self-limiting and usually resolves within 2 hours. b. The symptoms described are not associated with influenza. c. The symptoms described are not associated with urinary tract infection. d. Prolonged emptying time of bile from the gallbladder, combined with elevated blood cholesterol levels, make cholelithiasis a more probable diagnosis than indigestion. KEY: Integrated Process: Nursing Process: Clinical Problem Solving | Cognitive Level: Application | Content Area: Maternity | Client Need: Health Promotion and Maintenance | Difficulty Level: Moderate

During the postpartum assessment, the perinatal nurse notes that a patient who has just experienced a forceps-assisted birth now has a large quantity of bright red bleeding. Her uterine fundus is firm. The nurse's most appropriate action is to notify the physician/certified nurse midwife and describe a: a. Need for vaginal assessment and repair b. Requirement for an oxytocin infusion c. Need for further information for the woman/family about forceps d. Requirement for bladder assessment and catheterization

ANS: a In the presence of a firm fundus and bright red bleeding, after a forceps-assisted birth there is a need for vaginal assessment and there may be a need for repair.

An example of a cultural prescriptive belief during pregnancy is: a. Remain active during pregnancy b. Coldness in any form should be avoided c. Do not have your picture taken d. Avoid sexual intercourse during the third trimester

ANS: a The belief that the patient should remain active during pregnancy is the only example of a cultural prescriptive belief. All of the other answers are examples of cultural restrictive beliefs.

The perinatal nurse notes a rapid decrease in the fetal heart rate that does not recover immediately following an amniotomy. The most likely cause of this obstetrical emergency is: a. Prolapsed umbilical cord b. Vasa previa c. Oligohydramnios d. Placental abruption

ANS: a The nurse needs to assess the fetal heart rate immediately before and after the artificial rupture of membranes. Changes such as transient fetal tachycardia may occur and are common. However, other FHR patterns such as bradycardia and variable decelerations may be indicative of cord compression or prolapse.

The physician has ordered intravenous oxytocin for induction for four gravidas. In which of the following situations should the nurse refuse to comply with the order? a. Primigravida with complete placenta previa b. Multigravida with extrinsic asthma c. Primigravida who is 38 years old d. Multigravida who is colonized with group B streptococci

ANS: a The nurse should refuse to comply with this order because labor is contraindicated for a patient with complete placenta previa. This patient will have to be delivered via cesarean section.

1. A neonate is born at 33 weeks' gestation with a birth weight of 2400 grams. This neonate would be classified as: a. Low birth weight b. Very low birth weight c. Extremely low birth weight d. Very premature

ANS: a a. Neonates with a birth weight of less than 2500 grams but greater than 1500 grams are classified as low birth weight. b. Neonates with birth weight less than 1500 grams but greater than 1000 grams are classified as very low birth weight. c. Neonates with birth weight less than 1000 grams are classified as extremely low birth weight. d. Neonates born less than 32 weeks' gestation are classified as very premature.

44. Asking the pregnant woman about her use of recreational drugs is an essential component of the prenatal history. Harmful fetal effects that may occur from recreational drugs include (select all that apply): a. Miscarriage/spontaneous abortion b. Low birth weight c. Macrosomia d. Post-term labor/birth

ANS: a, b Illegal or recreational drug use can have a number of detrimental effects on maternal and fetal health, including spontaneous abortion, low birth weight, placental abruption, and preterm labor. KEY: Integrated Process: Nursing Process: Clinical Problem Solving | Cognitive Level: Application | Content Area: Maternity | Client Need: Safe and Effective Care Environment | Difficulty Level: Moderate

A G2 P1 woman who experienced a prolonged labor and prolonged rupture of membranes is at risk for metritis. Which of the following nursing actions are directed at decreasing this risk? (Select all that apply.) a. Instruct woman to increase her fluid intake b. Instruct woman to change her peri-pads after each voiding c. Instruct woman to ambulate in the halls four times a day d. Instruct woman to apply ice packs to the perineum

ANS: a, b, c a. Maintaining adequate hydration can decrease a person's risk for infection. b. Lochia is a media for bacterial growth, so it is important to frequently change the peri-pads. c. Ambulation can decrease the risk of infection by promoting uterine drainage. d. Ice pack therapy is directed at decreasing edema of the perineum and promoting comfort. It has no effect on metriosis.

20. Nursing actions that decrease the risk of skin breakdown include which of the following? (Select all that apply.) a. Using gelled mattresses b. Using emollients in groin and thigh areas c. Using transparent dressings d. Drying thoroughly

ANS: a, b, c a. Use of gelled mattresses decreases the risk of pressure sores. b. Use of emollients reduces the risk of irritation from urine. c. Use of transparent dressings reduces the risk of friction injuries. d. Drying thoroughly is important in maintaining body heat.

45. The clinic nurse schedules Tracy for her first prenatal appointment with the certified nurse-midwife (CNM) in the clinic. Tracy has appropriate questions for her potential health-care provider that include (select all that apply): a. Complementary and alternative methods used during labor and birth b. An opportunity to meet other providers in the practice c. Beliefs and practices concerning an episiotomy and an epidural anesthetic d. Whether the nurse-midwife will be continually available for support during labor

ANS: a, b, c A woman's journey through the pregnancy experience can have long-term effects on her self-perception and self-concept. Therefore, it is especially important that the patient choose a care provider and group with whom she can openly relate and who shares the same philosophical views on the management of pregnancy. At the first prenatal visit, it is not common to explore whether the nurse-midwife will be continually available for support during labor. KEY: Integrated Process: Nursing Process: Clinical Problem Solving | Cognitive Level: Application | Content Area: Peds/Maternity | Client Need: Safe and Effective Care Environment | Difficulty Level: Moderate

Documentation related to vacuum delivery includes which of the following: a. Fetal heart rate b. Timing and number of applications c. Position and station of fetal head d. Maternal position

ANS: a, b, c Assessment of fetal heart rate is part of second-stage management, timing and number of applications are part of standard of care related to safe vacuum deliveries, and position and station of fetal head are noted for safe vacuum extraction. Maternal position is not critical to the documentation related to vacuum deliveries.

47. An overweight or obese pre-pregnancy weight increases the risk for which poor maternal outcomes? (Select all that apply.) a. Preeclampsia b. Hemorrhage c. Difficult delivery d. Vaginal infections

ANS: a, b, c Being overweight or obese can substantially increase perinatal risk; however, no data support an increase in vaginal infections for the obese pregnant population. KEY: Integrated Process: Knowledge | Cognitive Level: Complication | Content Area: Maternity | Client Need: Health Promotion and Maintenance: Antepartum Care | Difficulty Level: Moderate

Hyperstimulation is defined as: a. Contractions lasting more than 2 minutes b. Five or more contractions in 10 minutes c. Contractions occurring within 1 minute of each other d. Uterine resting tone below 20 mm/Hg

ANS: a, b, c Contractions lasting more than 2 minutes, five or more contractions in 10 minutes, and contractions occurring within 1 minute of each other describe the criteria for hyperstimulation. Uterine resting tone below 20 mm/Hg reflects normal uterine resting tone.

40. The clinic nurse describes the respiratory system changes common to pregnancy to the new nurse. These changes include (select all that apply): a. An increased tidal volume b. A decreased airway resistance c. An increased chest circumference d. An increased airway resistance

ANS: a, b, c During pregnancy, a number of changes occur to meet the woman's increased oxygen requirements. The tidal volume (amount of air breathed in each minute) increases 30% to 40%. The enlarging uterus creates an upward pressure that elevates the diaphragm and increases the subcostal angle. The chest circumference may increase by as much as 6 centimeters, and airway resistance decreases. Although the "up and down" capacity of diaphragmatic movement is reduced, lateral movement of the chest and intercostal muscles accommodates for this loss of movement and keeps pulmonary functions stable. There is no increase in airway resistance during pregnancy. KEY: Integrated Process: Teaching and Learning | Cognitive Level: Knowledge | Content Area: Peds/Maternity | Client Need: Safe and Effective Care Environment | Difficulty Level: Easy

35. The clinic nurse discusses normal bladder function in pregnancy with a 22-year-old pregnant woman who is now in her 29th gestational week. The nurse explains that at this time in pregnancy, it is normal to experience (select all that apply): a. Urinary frequency b. Urinary urgency c. Nocturia d. Incontinence

ANS: a, b, c During pregnancy, the bladder, a pelvic organ, is compressed by the weight of the growing uterus. The added pressure, along with progesterone-induced relaxation of the urethra and sphincter musculature, leads to urinary urgency, frequency, and nocturia. Incontinence of urine is not a normal change during pregnancy. KEY: Integrated Process: Nursing Process: Clinical Problem Solving | Cognitive Level: Application | Content Area: Peds/Maternity | Client Need: Safe and Effective Care Environment | Difficulty Level: Moderate

42. The clinic nurse describes possible interventions for the pregnant woman who is experiencing pain and numbness in her wrists. The nurse suggests (select all that apply): a. Elevating the arms and wrists at night b. Reassessment during the postpartum period c. The use of "cock splints" to prevent wrist flexion d. Massaging the hands and wrists with alcohol

ANS: a, b, c Edema from vascular permeability can lead to a collection of fluid in the wrist that puts pressure on the median nerve lying beneath the carpal ligament, leading to carpal tunnel syndrome. Elevation of the hands at night may help to reduce the edema. Occasionally, a woman may need to wear a "cock splint" to prevent the wrist from flexing. Reassessment in the postpartum period is indicated because although carpal tunnel syndrome usually subsides after the pregnancy has ended, some women may require surgical treatment if symptoms persist. Massaging the hands and wrists with alcohol does not improve pain and numbness. KEY: Integrated Process: Teaching and Learning | Cognitive Level: Application | Content Area: Maternity | Client Need: Safe and Effective Care Environment | Difficulty Level: Moderate

The perinatal nurse screens all pregnant women early in pregnancy for maternal attachment risk factors, which include (select all that apply): a. Adolescence b. Low educational level c. History of depression d. A strong support system for the pregnancy

ANS: a, b, c Maternal attachment to the fetus is an important area to assess and can be useful in identifying families at risk for maladaptive behaviors. The nurse should assess for indicators such as unintended pregnancy, domestic violence, difficulties in the partner relationship, sexually transmitted infections, limited financial resources, substance use, adolescence, poor social support systems, low educational level, the presence of mental conditions, or adolescence that might interfere with the patient's ability to bond with and care for the infant. A strong support system can facilitate the patient's ability to bond with and care for the infant.

52. Jorgina is a 24-year-old pregnant woman at 26 weeks' gestation. This is Jorgina's third pregnancy, and her obstetrical history includes one full-term birth, one preterm birth, and two living children. Today Jorgina arrives at the clinic with complaints of fatigue, insomnia, and backache. She reports that she is a nurse on an oncology unit and is worried about continuing with working her 12-hour shifts. The perinatal nurse identifies concerns in Jorgina's history and work environment including (select all that apply): a. Risk of preterm birth b. Presence of chemotherapeutic agents c. Requirement for heavy lifting d. History of diabetes

ANS: a, b, c Women who are currently experiencing pregnancy complications and those who have a history of pregnancy complications (such as history of preterm birth) or other preexisting health disorders may be required to reduce their hours or stop working. The potential for maternal exposure to toxic substances such as chemotherapeutic agents, lead, and ionizing radiation (found in laboratories and health-care facilities); heavy lifting; and use of heavy machinery and other hazardous equipment should prompt reassignment to a different work area. If reassignment is not possible, Jorgina may need to stop working until the pregnancy has been completed. In this scenario there is no history of diabetes. KEY: Integrated Process: Nursing Process: Clinical Problem Solving | Cognitive Level: Application | Content Area: Maternity | Client Need: Safe and Effective Care Environment | Difficulty Level: Moderate

25. A nurse is completing the initial assessment on a neonate of a mother with type I diabetes. Important assessment areas for this neonate include which of the following? (Select all that apply.) a. Assessment of cardiovascular system b. Assessment of respiratory system c. Assessment of musculoskeletal system d. Assessment of neurological system

ANS: a, b, c, d a. Neonates of mothers with type I diabetes are at higher risk for cardiac anomalies. b. Neonates of mothers with type I diabetes are at higher risk for RDS due to a delay in surfactant production related to high maternal glucose levels. c. Neonates of mothers with type I diabetes are usually large and are at risk for a fractured clavicle. d. Neonates of mothers with type I diabetes are at higher risk for neurological damage and seizures due to neonatal hyperinsulinism.

34. The perinatal nurse teaches the student nurse about the physiological changes in pregnancy that most often contribute to the increased incidence of urinary tract infections. These changes include (select all that apply): a. Relaxation of the smooth muscle of the urinary sphincter b. Relaxation of the smooth muscle of the bladder c. Inadequate emptying of the bladder d. Increased incidence of bacteriuria

ANS: a, b, c, d Ascension of bacteria into the bladder can cause asymptomatic bacteriuria (ASB), or urinary tract infections (UTIs). These infections occur more frequently in pregnancy due to relaxation of the smooth muscle of the bladder and urinary sphincter and inadequate emptying of the bladder, changes that allow bacterial ascent into the bladder. KEY: Integrated Process: Nursing Process: Clinical Problem Solving | Cognitive Level: Application | Content Area: Peds/Maternity | Client Need: Safe and Effective Care Environment | Difficulty Level: Moderate

38. The clinic nurse encourages all pregnant women to increase their water intake to at least 8 to 10 glasses per day in order to (select all that apply): a. Decrease the risk of constipation b. Decrease the risk of bile stasis c. Decrease their feelings of fatigue d. Decrease the risk of urinary tract infections

ANS: a, b, c, d Patients should be encouraged to drink at least 8 to 10 glasses of water each day and empty their bladders at least every 2 to 3 hours and immediately after intercourse. These measures will help prevent stasis of urine and the bacterial contamination that leads to infection, as well as constipation. Some women experience symptoms of fatigue that can be alleviated by remaining adequately hydrated. KEY: Integrated Process: Teaching and Learning | Cognitive Level: Application | Content Area: Maternity | Client Need: Safe and Effective Care Environment | Difficulty Level: Moderate

24. A nurse is caring for a 40 weeks' gestation neonate. The neonate is 12 hours post-birth and has been admitted to the NICU for meconium aspiration. The nurse recalls that the following are potential complications related to meconium aspiration (select all that apply): a. Obstructed airway b. Hyperinflation of the alveoli c. Hypoinflation of the alveoli d. Decreased surfactant proteins

ANS: a, b, d a. The presence of meconium in the neonate's lungs can cause a partial obstruction of the lower airway that leads to a trapping of air and a hyperinflation of the alveoli. b. The presence of meconium in the neonate's lungs can cause a partial obstruction of the lower airway that leads to a trapping of air and a hyperinflation of the alveoli. c. The presence of meconium in the neonate's lungs can cause a partial obstruction of the lower airway that leads to a trapping of air and a hyperinflation of the alveoli. d. The presence of meconium in the lungs can also cause a chemical pneumonitis and inhibit surfactant production.

51. Interventions for low back pain during pregnancy should include (select all that apply): a. Utilizing proper body mechanics b. Applying ice or heat to affected area c. Avoiding pelvic rock and pelvic tilt d. Using additional pillows for support during sleep

ANS: a, b, d Interventions for back pain during pregnancy include utilizing proper body mechanics, applying heat or ice to the area, using additional pillows during sleep, and not avoiding pelvic rock/tilt, but encouraging pelvic rock/tilt. KEY: Integrated Process: Teaching and Learning | Cognitive Level: Knowledge | Content Area: Intrapartum care | Client Need: Health Promotion and Maintenance: Intrapartum Care | Difficulty Level: Moderate

39. The perinatal nurse examines the thyroid gland as part of the physical examination of Savannah, a pregnant woman who is now at 16 weeks' gestation. The perinatal nurse informs Savannah that during pregnancy (select all that apply): a. Increased size of the thyroid gland is normal b. Increased function of the thyroid gland is normal c. Decreased function of the thyroid gland is normal d. The thyroid gland will return to its normal size and function during the postpartal period

ANS: a, b, d The thyroid gland changes in size and activity during pregnancy. Enlargement is caused by increased circulation from the progesterone-induced effects on the vessel walls, and by estrogen-induced hyperplasia of the glandular tissue. The thyroid gland increases not decreases in size and activity during pregnancy. The thyroid gland returns to normal size and activity postpartum. KEY: Integrated Process: Teaching and Learning | Cognitive Level: Application | Content Area: Maternity | Client Need: Safe and Effective Care Environment | Difficulty Level: Moderate

18. A nurse is caring for a 2-day-old neonate who was born at 31 weeks' gestation. The neonate has a diagnosis of respiratory distress syndrome (RDS). Which of the following medical treatments would the nurse anticipate for this neonate? (Select all that apply.) a. Exogenous surfactant b. Corticosteroids c. Continuous positive airway pressure (CPAP) d. Bronchodilators

ANS: a, c a. This is a common medical treatment for RDS. b. Corticosteroids are given to women in preterm labor to decrease the risk of RDS. c. CPAP is used to assist neonates with RDS. d. Bronchodilators are given to neonates with bronchopulmonary dysplasia (BPD).

54. Teera is a 22-year-old woman who is experiencing her third pregnancy. Her obstetrical history includes one first-trimester elective abortion and one first-trimester spontaneous abortion. Teera is a semi-vegetarian who drinks milk and eats yogurt and fish as part of her daily intake. The perinatal nurse discusses Teera's diet with her as she may be deficient in (select all that apply): a. Iron b. Magnesium c. Zinc d. Vitamin B12

ANS: a, c Semi-vegetarian diets include fish, poultry, eggs, and dairy products but no beef or pork and have adequate intake of magnesium. Pregnant women who adhere to this diet may consume inadequate amounts of iron and zinc. Because strict vegetarians (vegans) consume only plant products, their diets are deficient in vitamin B12, found only in foods of animal origin. KEY: Integrated Process: Teaching and Learning | Cognitive Level: Application | Content Area: Maternity | Client Need: Safe and Effective Care Environment | Difficulty Level: Moderate

53. The clinic nurse is assessing the complete blood count results for Kim-Ly, a 23-year-old pregnant woman. Kim-Ly's hemoglobin is 9.8 g/dL. This laboratory finding places Kim-Ly's pregnancy at risk for (select all that apply): a. Preterm birth b. Placental abruption c. Intrauterine growth restriction d. Thrombocytopenia

ANS: a, c True anemia, or iron-deficiency anemia, occurs when the hemoglobin level drops below 10 g/dL. The blood's decreased oxygen-carrying capacity causes a reduction in oxygen transport to the developing fetus. Decreased fetal oxygen transport has been associated with intrauterine growth restriction (IUGR) and preterm birth. There is not a risk factor for abruption or thrombocytopenia. KEY: Integrated Process: Nursing Process: Clinical Problem Solving | Cognitive Level: Analysis | Content Area: Maternity | Client Need: Safe and Effective Care Environment | Difficulty Level: Difficult

22. A nurse is caring for a 10-day-old neonate who was born at 33 weeks' gestation. Which of the following actions assist the nurse in assessing for signs of feeding tolerance? (Select all that apply.) a. Check for presence of bowel sounds b. Assess temperature c. Check gastric residual by aspirating stomach contents d. Assess stools

ANS: a, c, d a. Feedings should be held and physician notified if bowel sounds are absent. b. The neonate's temperature has no direct effect on feeding tolerance. c. Aspirated stomach contents are assessed for amount, color, and consistency. This assists in the evaluation of the degree of digestion and absorption. d. Stools are assessed for consistency, amount, and frequency. This assists in the evaluation of the degree of digestion and absorption.

41. The clinic nurse teaches the new nurse about pregnancy-induced blood clotting changes. The nurse explains that a pregnant woman is at risk for venous thrombosis due to (select all that apply): a. Increased fibrinogen volume b. Increased blood factor V c. Increased blood factor X d. Venous stasis

ANS: a, c, d Although the platelet cell count does not change significantly during pregnancy, fibrinogen volume has been shown to increase by as much as 50%. This alteration leads to an increase in the sedimentation rate. Blood factors VII, VIII, IX, and X are also increased, and this change causes hypercoagulability. The hypercoagulability state, coupled with venous stasis (poor blood return from the lower extremities) places the pregnant woman at an increased risk for venous thrombosis, embolism, and, when complications are present, disseminated intravascular coagulation (DIC). Blood factor V does not increase. KEY: Integrated Process: Teaching and Learning | Cognitive Level: Application | Content Area: Maternity | Client Need: Safe and Effective Care Environment | Difficulty Level: Moderate

Contraindications for induction of labor include: a. Abnormal fetal position b. Postdated pregnancy c. Pregnancy-induced hypertension d. Placental abnormalities

ANS: a, d Contraindications for induction of labor include abnormal fetal position because of the risk of fetal injury and placental abnormalities because of the risk of hemorrhage. Pregnancy-induced hypertension and placental abnormalities are two of the common indications for induction of labor.

43. The clinic nurse advocates for smoking cessation during pregnancy. Potential harmful effects of prenatal tobacco use include (select all that apply): a. Preterm birth b. Gestational hypertension c. Gestational diabetes d. Low birth weight

ANS: a, d Nurses can help to improve the fetal environment by educating women about the dangers of direct and passive smoking during pregnancy. Effects of tobacco use during pregnancy are well documented and predispose to premature rupture of the membranes, preterm labor, placental abruption, placenta previa, and infants who are low birth weight or small for gestational age (SGA). Gestational hypertension and diabetes are not associated with smoking during pregnancy. KEY: Integrated Process: Nursing Process: Clinical Problem Solving | Cognitive Level: Knowledge | Content Area: Maternity | Client Need: Health Promotion and Maintenance | Difficulty Level: Easy

5. The perinatal nurse teaches the postpartum woman about warning signs regarding development of postpartum infection. Signs and symptoms that merit assessment by the health-care provider include the development of a fever and: a. Breast engorgement b. Uterine tenderness c. Diarrhea d. Emotional lability

ANS: b During the immediate postpartum period, the most common site of infection is the uterine endometrium. This infection presents with a temperature elevation over 101°F, often within the first 24 to 48 hours after childbirth, followed by uterine tenderness and foul-smelling lochia.

14. Which of the following neonatal signs or symptoms would the nurse expect to see in a neonate with an elevated bilirubin level? a. Low glucose b. Poor feeding c. Hyperactivity d. Hyperthermia

ANS: b a. Hypoglycemia is not a sign that is related to an elevated bilirubin level. b. The baby is likely to feed poorly. An elevated bilirubin level adversely affects the central nervous system. Babies are often sleepy and feed poorly when the bilirubin level is elevated. c. Hyperactivity is the opposite of the behavior one would expect the baby to exhibit. d. Hyperthermia is not directly related to an elevated bilirubin level.

Cathy is pregnant for the second time. Her son, Steven, has just turned 2 years old. She asks you what she should do to help him get ready for the expected birth. What is the nurse's most appropriate response? a. Steven will probably not understand any explanations about the arrival of the new baby, so Cathy should do nothing. b. If Steven's sleeping arrangements need to be changed, it should be done well in advance of the birth. c. Steven should come to the next prenatal visit and listen to the fetal heartbeat to encourage sibling attachment. d. Steven should be encouraged to plan an elaborate welcome for the newborn.

ANS: b Children still sleeping in a crib should be moved to a bed at least 2 months before the baby is due, as this age group is particularly sensitive to disruptions of the physical environment.

Which of the following would be a priority for the nurse when caring for a pregnant woman who has recently emigrated from another country? a. Help her develop a realistic, detailed birth plan. b. Identify her support system. c. Teach her about expected emotional changes of pregnancy. d. Refer her to a doula for labor support.

ANS: b Correct, because lack of social support has been correlated with an increased risk of pregnancy complications and difficult adaptation to pregnancy. Pregnant women who are recent immigrants face many challenges in obtaining needed social support, and the nurse should first identify her support system to plan further interventions and referrals.

A pregnant client asks the nurse why she should attend childbirth classes. The nurse's response would be based on which of the following information? a. Attending childbirth class is a good way to make new friends. b. Childbirth classes will help new families develop skills to meet the challenges of childbirth and parenting. c. Attending childbirth classes will help a pregnant woman have a shorter labor. d. Childbirth classes will help a pregnant woman decrease her chance of having a cesarean delivery.

ANS: b Correct. These are the stated goals of childbirth education (ICEA, Lamaze).

28. The nurse notes each of the following findings in a woman at 10 weeks' gestation. Which of the findings would enable the nurse to tell the woman that she is probably pregnant? a. Fetal heart rate via Doppler b. Positive pregnancy test c. Positive ultrasound assessment d. Absence of menstrual period

ANS: b Feedback a. A fetal heart rate is a positive sign of pregnancy. b. A positive pregnancy test is a probable sign of pregnancy. It is not a positive sign because the hormone tested for—human chorionic gonadatropin (hCG)—may be being produced by, for example, a hydatidiform mole. c. A positive ultrasound is a positive sign of pregnancy. d. Amenorrhea is a presumptive sign of pregnancy. KEY: Integrated Process: Nursing Process: Analysis | Cognitive Level: Comprehension | Content Area: Antepartum Care | Client Need: Health Promotion and Maintenance | Difficulty Level: Easy

7. At the end of her 32-week prenatal visit, a woman reports discomfort with intercourse and tells you shyly that she wants to maintain a sexual relationship with her partner. The best response is to: a. Reassure woman/couple of normalcy of response b. Suggest alternative positions for sexual intercourse and alternative sexual activity to sexual intercourse c. Recommend cessation of intercourse until after delivery due to advanced gestation d. Suggest woman discuss this with her care provider at her next appointment

ANS: b Feedback a. Although this is a normal response, providing reassurance is not enough. Further intervention is indicated. b. Although shy to discuss this, she wants to maintain a sexual relationship with her partner. Suggesting alternative positions for sexual intercourse and alternative sexual activity to sexual intercourse provides the woman with information to maintain sexual relations. c. She wants to maintain a sexual relationship with her partner, and there are no contraindications to intercourse during a healthy pregnancy. d. The patient is seeking out information and to defer her to her care provider at her next appointment is inappropriate. Additionally, she may not be comfortable discussing this with anyone else. KEY: Integrated Process: Clinical Problem Solving | Cognitive Level: Complication | Content Area: Maternity | Client Need: Health Promotion and Maintenance | Difficulty Level: Moderate

30. The nurse who is assessing a G2 P1 palpates the fundal height at the location noted on the picture below. The nurse concludes that the fetus is equal to which of the following gestational ages? a. 12 weeks b. 20 weeks c. 28 weeks d. 36 weeks

ANS: b Feedback a. At 12 weeks' gestation, the fundus should be felt at the level of the symphysis pubis. b. The fundus at the level of the umbilicus indicates 20 weeks' gestation. In this question, the fact that this patient is a multigravida is not relevant. Uterine growth should be consistent for both primigravidas and multigravidas. c. At 28 weeks' gestation, the fundus should be felt 8 cm above the level of the umbilicus. d. At 36 weeks' gestation, the fundus should be felt at the xiphoid process. KEY: Integrated Process: Nursing Process: Analysis; Nursing Process: Assessment | Cognitive Level: Application | Content Area: Antepartum Care | Client Need: Health Promotion and Maintenance | Difficulty Level: Easy

24. Which of the following findings, seen in pregnant women in the third trimester, would the nurse consider to be within normal limits? a. Diplopia b. Epistaxis c. Bradycardia d. Oliguria

ANS: b Feedback a. Diplopia is sometimes seen in patients with pregnancy-induced hypertension (PIH). b. Epistaxis is commonly seen in pregnant patients. The bleeding is related to the increased vascularity of the mucous membranes. Unless the blood loss is significant, it is a normal finding. c. Bradycardia is often seen immediately after delivery but not during the third trimester. d. Oliguria is seen in patients with PIH. KEY: Integrated Process: Nursing Process: Analysis | Cognitive Level: Application | Content Area: Antepartum Care | Client Need: Health Promotion and Maintenance | Difficulty Level: Easy

15. A woman presents to a prenatal clinic appointment at 10 weeks' gestation, in the first trimester of pregnancy. Which of the following symptoms would be considered a normal finding at this point in pregnancy? a. Occipital headache b. Urinary frequency c. Diarrhea d. Leg cramps

ANS: b Feedback a. Headaches may be benign or, especially if noted after 20 weeks' gestation, may be a symptom of pregnancy-induced hypertension (PIH). b. Urinary frequency is a common complaint of women during their first trimester. c. Diarrhea is rarely seen in pregnancy. Constipation is a common complaint. d. Leg cramps are commonly seen during the second and third trimesters. KEY: Integrated Process: Nursing Process: Assessment | Cognitive Level: Comprehension | Content Area: Antepartum Care | Client Need: Health Promotion and Maintenance | Difficulty Level: Easy

5. Intimate partner violence (IPV) against women consists of actual or threatened physical or sexual violence and psychological and emotional abuse. Screening for IPV during pregnancy is recommended for: a. Pregnant women with a history of domestic violence b. All pregnant women c. All low-income pregnant women d. Pregnant adolescents

ANS: b Feedback a. Intimate partner violence is underreported by women, necessitating universal screening. b. Correct. AWHONN advocates for universal screening for domestic violence for all pregnant women. Homicide is the most likely cause of death for pregnant or recently pregnant women, and a significant portion of those homicides are committed by their intimate partners. One in six pregnant women reported physical or sexual abuse during pregnancy, seriously impacting maternal and fetal health and infant birth weight. c. IPV crosses all ethnic, racial, religious, and socioeconomic levels. d. IPV crosses all ethnic, racial, religious, and socioeconomic levels. KEY: Integrated Process: Clinical Problem Solving | Cognitive Level: Application | Content Area: Maternity | Client Need: Psychosocial Integrity | Difficulty Level: Moderate

13. The clinic nurse talks with Kathy about her possible pregnancy. Kathy has experienced amenorrhea for 2 months, nausea during the day with vomiting every other morning, and breast tenderness. These symptoms are best described as: a. Positive signs of pregnancy b. Presumptive signs of pregnancy c. Probable signs of pregnancy d. Possible signs of pregnancy

ANS: b Feedback a. Positive signs include fetal heartbeat, visualization of the fetus, and fetal movements palpated by the examiner. b. Presumptive signs of pregnancy include amenorrhea, nausea and vomiting, frequent urination, breast tenderness, perception of fetal movement, skin changes, and fatigue. Probable signs of pregnancy include abdominal enlargement, Piskacek sign, Hegar sign, Goodell sign, Braxton Hicks sign, positive pregnancy test, and ballottement. Positive signs include fetal heartbeat, visualization of the fetus, and fetal movements palpated by the examiner. c. Probable signs of pregnancy include abdominal enlargement, Piskacek sign, Hegar sign, Goodell sign, Braxton Hicks sign, positive pregnancy test, and ballottement. d. Possible signs of pregnancy may vary widely. KEY: Integrated Process: Nursing Process: Clinical Problem Solving | Cognitive Level: Application | Content Area: Maternity | Client Need: Safe and Effective Care Environment | Difficulty Level: Moderate

23. A pregnant woman informs the nurse that her last normal menstrual period was on July 6, 2007. Using Naegele's rule, which of the following would the nurse determine to be the patient's estimated date of delivery (EDC)? a. January 9, 2008 b. April 13, 2008 c. April 20, 2008 d. September 6, 2008

ANS: b Feedback a. The EDC is calculated as April 13, 2008. b. The EDC is calculated as April 13, 2008. Naegele's rule: subtract 3 months and add 7 days to the first day of the last normal menstrual period. c. The EDC is calculated as April 13, 2008. d. The EDC is calculated as April 13, 2008. KEY: Integrated Process: Nursing Process: Assessment | Cognitive Level: Application | Content Area: Antepartum Care | Client Need: Health Promotion and Maintenance | Difficulty Level: Easy

10. The clinic nurse reviews the complete blood count results for a 30-year-old woman who is now 33 weeks' gestation. Tamara's hemoglobin value is 11.2 g/dL, and her hematocrit is 38%. The clinic nurse interprets these findings as: a. Normal adult values b. Normal pregnancy values for the third trimester c. Increased adult values d. Increased values for 33 weeks' gestation

ANS: b Feedback a. The values are low normal for adults but represent normal findings for pregnant women. b. During pregnancy the woman's hematocrit values may appear low due to the increase in total plasma volume (on average, 50%). Because the plasma volume is greater than the increase in erythrocytes (30%), the hematocrit decreases by about 7%. This alteration is termed "physiologic anemia of pregnancy," or "pseudo-anemia." The hemodilution effect is most apparent at 32 to 34 weeks. The mean acceptable hemoglobin level in pregnancy is 11 to 12 g/dL of blood. c. The values are not increased; they are low normal for adults but represent normal findings for pregnant women. d. The values are not increased; they are low normal for adults but represent normal findings for pregnant women. KEY: Integrated Process: Nursing Process: Clinical Problem Solving | Cognitive Level: Analysis | Content Area: Peds/Maternity | Client Need: Safe and Effective Care Environment | Difficulty Level: Difficult

Your patient is a 28-year-old gravida 2 para 1 in active labor. She has been in labor for 12 hours. Upon further assessment, the nurse determines that she is experiencing a hypotonic labor pattern. Possible maternal and fetal implications from hypotonic labor patterns are: a. Intrauterine infection and maternal exhaustion with fetal distress usually occurring early in labor. b. Intrauterine infection and maternal exhaustion with fetal distress usually occurring late in labor. c. Intrauterine infection and postpartum hemorrhage with fetal distress early in labor. d. Intrauterine infection and ruptured uterus and fetal death.

ANS: b Hypotonic labor patterns increase risk for infection and maternal exhaustion, with fetal distress occurring late in labor as hypotonic patterns prolong labor.

The clinic nurse visits with Wayne, a 32-year-old man whose partner is pregnant for the first time and is at 12 weeks. Wayne describes nausea and vomiting, fatigue, and weight gain. His symptoms are best described as: a. Influenza b. Couvade syndrome c. Acid reflux d. Cholelithiasis

ANS: b In preparation for parenthood, the male partner moves through a series of developmental tasks. During the first trimester, the father begins to deal with the reality of the pregnancy and may worry about financial strain and his ability to be a good father. Feelings of confusion and guilt often surface with the recognition that he is not as excited about the pregnancy as his partner, and couvade syndrome, the experience of maternal signs and symptoms, may develop.

A primigravida woman at 42 weeks' gestation received Prepidil (dinoprostone) for induction 12 hours ago. The Bishop score is now 3. Which of the following actions by the nurse is appropriate? a. Perform Nitrazine analysis of the amniotic fluid. b. Report the lack of progress to the obstetrician. c. Place the woman on her left side. d. Ask the doctor for an order for oxytocin.

ANS: b Little progress has taken place. The Bishop score of a primigravida will need to be 9 or higher before oxytocin will be effective.

You are caring for a primiparous woman admitted to labor and delivery for induction of labor at 42 weeks' gestation. She asks you to explain the factors that contribute to prolonged labor. The best response would be to state the following: a. Primiparous women are not at risk for dystocia because they usually have small babies. b. Dystocia is related to uterine contractions, the pelvis, the fetus, the position of the mother, and psychosocial response. c. Labor is primarily associated with pelvic abnormalities. d. Dystocia is typically diagnosed prior to labor based on pelvimetry.

ANS: b This is the only correct definition of prolonged labor and dystocia. The success of any labor depends on the complex interrelationship of several factors: fetal size, presentation, position, size and shape of the pelvis, and quality of uterine contractions.

27. The perinatal nurse caring for Emily, a 24-year-old mother of an infant born at 26 weeks' gestation, is providing discharge teaching. Emily is going to travel to the specialty center approximately 200 miles away where her daughter is receiving care. The nurse tells Emily that it is normal for Emily to feel (select all that apply): a. In control b. Anxious c. Guilty d. Overwhelmed

ANS: b, c, d a. Parents usually feel out of control. b. Correct answer. c. Correct answer. d. Correct answer.

36. A 32-year-old woman now at 32 weeks' gestation is complaining of right-sided sharp abdominal pain. The patient is examined by the clinic nurse and given information about abdominal discomfort in pregnancy. She is also instructed to seek immediate attention if she (select all that apply): a. Has heartburn b. Has chills or a fever c. Feels decreased fetal movements d. Has increased abdominal pain

ANS: b, c, d Heartburn is a common discomfort throughout pregnancy. Because the appendix is pushed upward and posterior by the gravid uterus, the typical location of pain is not a reliable indicator for a ruptured appendix during pregnancy. The pain should gradually subside, but if it persists or is accompanied by fever, a change in bowel habits, or decreased fetal movement, the patient should promptly contact her medical provider. KEY: Integrated Process: Teaching and Learning | Cognitive Level: Application | Content Area: Maternity | Client Need: Safe and Effective Care Environment | Difficulty Level: Moderate

The clinic nurse encourages paternal attachment during pregnancy by including the father in (select all that apply): a. Prenatal visits b. Ultrasound appointments c. Prenatal class information d. History taking and obtaining prenatal screening information

ANS: b, c, d Pregnancy is psychologically stressful for men; some enjoy the role of nurturer, but others feel alienated and begin to stray from the relationship. The nurse can be instrumental in promoting early paternal attachment. Involvement of the father during examinations and tests and prenatal classes, along with thorough explanations of the need for them, can minimize the father's feelings of being left out. A history and prenatal screening should be conducted at the first prenatal visit with the woman alone to ensure confidentiality and an open discussion of any problems or concerns she may have. The history should include information about the current pregnancy; the obstetric and gynecologic history; and a cultural assessment, and a medical, nutritional, social, and family (including the father's) medical history.

73. The clinic nurse is aware of the importance of chlamydia screening during pregnancy. Chlamydia transmission to the infant at __________ may result in __________.

ANS: birth; ophthalmia neonatorum Chlamydia trachomatis is a bacteria that causes infection that is prevalent in sexually active populations, especially those in the under-25 age group. Complications of chlamydia infections include salpingitis, pelvic inflammatory disease, infertility, ectopic pregnancy, premature rupture of the membranes, and preterm birth. Transmission to the neonate may occur during birth and results in ophthalmia neonatorum and chlamydial neonatal pneumonia. KEY: Integrated Process: Nursing Process: Clinical Problem Solving | Cognitive Level: Knowledge | Content Area: Peds/Maternity | Client Need: Health Promotion and Maintenance | Difficulty Level: Easy

10. The perinatal nurse notifies the physician of the findings related to Juanita's assessment. The first step in care will most likely be to: a. Prepare Juanita for surgery b. Administer intravenous fluids c. Apply ice to the perineum d. Insert a urinary catheter

ANS: c If the hematoma is less than 3 to 5 centimeters in diameter, the physician usually orders palliative treatments such as ice to the area for the first 12 hours along with pain medication. After 12 hours, sitz baths are prescribed to replace the application of ice. However, a hematoma larger than 5 centimeters may require incision and drainage with the possible placement of a drain.

8. The nurse is assessing a baby girl on admission to the newborn nursery. Which of the following findings should the nurse report to the neonatologist? a. Intermittent strabismus b. Startling c. Grunting d. Vaginal bleeding

ANS: c a. Pseudostrabismus is a normal finding. b. Startling is a normal finding. c. Grunting is a sign of respiratory distress. The neonatologist should be notified. d. Vaginal bleeding is a normal finding.

The perinatal nurse is providing care to Carol, a 28-year-old multiparous woman in labor. Upon arrival to the birthing suite, Carol was 7 cm dilated and experiencing contractions every 1 to 2 minutes which she describes as "strong." Carol states she labored for 1 hour at home. As the nurse assists Carol from the assessment area to her labor and birth room, Carol states that she is feeling some rectal pressure. Carol is most likely experiencing: a. Hypertonic contractions b. Hypotonic contractions c. Precipitous labor d. Uterine hyperstimulation

ANS: c Contrary to both hypertonic and hypotonic labor, precipitate labor contractions produce very rapid, intense contractions. A precipitous labor lasts less than 3 hours from the beginning of contractions to birth. Patients often progress through the first stage of labor with little or no pain and may present to the birth setting already advanced into the second stage of labor.

Sally is in her third trimester and has begun to sing and talk to the fetus. Sally is probably exhibiting signs of: a. Mental illness b. Delusions c. Attachment d. Crisis

ANS: c Correct, because talking to the fetus is a sign of positive maternal adaptation. All other answers indicate pathology.

During labor induction with oxytocin, the fetal heart rate baseline is in the 140s with moderate variability. Contraction frequency is assessed to be every 2 minutes with duration of 60 seconds, of moderate strength to palpation. Based on this assessment, the nurse should take which action? a. Increase oxytocin infusion rate per physician's protocol. b. Stop oxytocin infusion immediately. c. Maintain present oxytocin infusion rate and continue to assess. d. Decrease oxytocin infusion rate by 2 mU/min and report to physician.

ANS: c Correct. Maintain present oxytocin infusion rate and continue to assess is the correct response, as this question describes a normal uterine contraction pattern.

Which statement best exemplifies adaptation to pregnancy in relation to the adolescent? a. Adolescents adapt to motherhood in a similar way to other childbearing women. b. Social support has very little effect on adolescent adaptation to pregnancy. c. The pregnant adolescent faces the challenge of multiple developmental tasks. d. Pregnant adolescents of all ages can be capable and active participants in health-care decisions.

ANS: c Correct. Pregnant adolescents face conflicting and multiple developmental tasks of pregnancy and adolescence at the same time.

19. The nurse is working in a prenatal clinic caring for a patient at 14 weeks' gestation, G2 P1001. Which of the following findings should the nurse highlight for the nurse midwife? a. Body mass index of 23 b. Blood pressure of 100/60 c. Hematocrit of 29% d. Pulse rate of 76 bpm

ANS: c Feedback a. A body mass index of 23 is normal. b. A blood pressure of 100/60 is normal. c. A hematocrit of 29% indicates that the patient is anemic. The nurse should highlight the finding for the nurse-midwife. d. A pulse rate of 76 bpm is a normal rate. KEY: Integrated Process: Nursing Process: Implementation | Cognitive Level: Application | Content Area: Antepartum Care; Potential for Alterations in Body Systems; Reduction of Risk Potential: Laboratory Values | Client Need: Health Promotion and Maintenance: Antepartum Care; Physiological Integrity: Reduction of Risk Potential | Difficulty Level: Easy

31. A patient at 28 weeks' gestation was last seen in the prenatal clinic at 24 weeks' gestation. Which of the following changes should the nurse bring to the attention of the Certified Nurse Midwife? a. Weight change from 128 pounds to 132 pounds b. Pulse change from 88 bpm to 92 bpm c. Blood pressure change from 110/70 to 140/90 d. Respiratory change from 16 rpm to 20 rpm

ANS: c Feedback a. A weight change of approximately 4 pounds in 4 weeks is normal in the second and third trimesters of pregnancy. b. This pulse rate change is within normal limits. c. A blood pressure elevation to 140/90 is a sign of mild preeclampsia. d. This respiratory rate change is within normal limits. KEY: Integrated Process: Nursing Process: Analysis; Nursing Process: Implementation | Cognitive Level: Application | Content Area: Antepartum Care; Reduction of Risk Potential—Potential for Alterations in Body Systems | Client Need: Health Promotion and Maintenance; Physiological Integrity: Reduction of Risk Potential | Difficulty Level: Easy

11. The clinic nurse is aware that the pregnant woman's blood volume increases by: a. 20% to 25% b. 30% to 35% c. 40% to 45% d. 50% to 55%

ANS: c Feedback a. An increase in maternal blood volume begins during the first trimester and peaks at term. The increase approaches 40% to 45%, not 20% to 25%. b. An increase in maternal blood volume begins during the first trimester and peaks at term. The increase approaches 40% to 45, not 30% to 35%. c. An increase in maternal blood volume begins during the first trimester and peaks at term. The increase approaches 40% to 45% and is primarily due to an increase in plasma and erythrocyte volume. Additional erythrocytes, needed because of the extra oxygen requirements of the maternal and placental tissue, ensure an adequate supply of oxygen to the fetus. The elevation in erythrocyte volume remains constant during pregnancy. d. An increase in maternal blood volume begins during the first trimester and peaks at term. The increase approaches 40% to 45%, not as high as 50% to 55%. KEY: Integrated Process: Nursing Process: Clinical Problem Solving | Cognitive Level: Knowledge | Content Area: Peds/Maternity | Client Need: Safe and Effective Care Environment | Difficulty Level: Easy

25. A primigravida patient is 39 weeks pregnant. Which of the following symptoms would the nurse expect the patient to exhibit? a. Nausea b. Dysuria c. Urinary frequency d. Intermittent diarrhea

ANS: c Feedback a. Nausea is usually not seen in the third trimester. b. Dysuria is not a normal finding at any time during a pregnancy. The possibility of a urinary traction infection (UTI) should be considered. c. Urinary frequency recurs at the end of the third trimester. As the uterus enlarges, it again compresses the bladder causing urinary frequency. d. Diarrhea is not a normal finding at any time during a pregnancy. KEY: Integrated Process: Nursing Process: Assessment | Cognitive Level: Application | Content Area: Antepartum Care | Client Need: Health Promotion and Maintenance | Difficulty Level: Easy

26. The nurse has taken a health history on four multigravida patients at their first prenatal visits. It is high priority that the patient whose first child was diagnosed with which of the following diseases receives nutrition counseling? a. Development dysplasia of the hip b. Achondroplastic dwarfism c. Spina bifida d. Muscular dystrophy

ANS: c Feedback a. The etiology of developmental dysplasia of the hip is unrelated to the mother's nutritional status. b. Achondroplasia is an inherited defect. Its etiology is unrelated to the mother's nutritional status. c. The incidence of spina bifida is much higher in women with poor folic acid intakes. It is a priority that this patient receives nutrition counseling. d. Most forms of muscular dystrophy are inherited. Their etiologies are unrelated to the mother's nutritional status. KEY: Integrated Process: Nursing Process: Implementation | Cognitive Level: Application | Content Area: Antepartum Care; Collaboration with Interdisciplinary Team; Management of Care: Referrals | Client Need: Health Promotion and Maintenance; Safe and Effective Care Environment: Management of Care | Difficulty Level: Moderate

22. A nurse is performing an assessment on a pregnant woman during a prenatal visit. Which of the following findings would lead the nurse to report to the obstetrician that the patient may be experiencing intrauterine growth restriction (IUGR)? a. Leopold's maneuvers: Hard round object in the fundus, flat object on left of uterus, small parts on right of uterus, soft round object above the symphysis b. Weight gain: 6-pound increase over 4-week period c. Fundal height measurement: 22 cm at 26 weeks' gestation d. Alpha-fetoprotein assessment: level is one-half normal, accompanied by complaints of severe nausea and vomiting

ANS: c Feedback a. This baby is in the breech position. This is not a sign of IUGR. b. This weight gain is slightly above normal. This is not a sign of IUGR. c. The fundal height at 26 weeks should be approximately 26 cm. The fundal height, therefore, is below expected. This patient may be experiencing intrauterine growth restriction. d. A low AFP level is seen in patients whose babies have spina bifida and other central nervous system defects. KEY: Integrated Process: Nursing Process: Analysis | Cognitive Level: Application Content Area: Antepartum Care | Client Need: Health Promotion and Maintenance | Difficulty Level: Moderate

3. During a routine prenatal visit in the third trimester, a woman reports she is dizzy and lightheaded when she is lying on her back. The most appropriate nursing action would be to: a. Order an EKG. b. Report this abnormal finding immediately to her care provider. c. Teach the woman to avoid lying on her back and to rise slowly because of supine hypotension. d. Order a nonstress test to assess fetal well-being.

ANS: c Feedback a. This is a normal occurrence in pregnancy and does not indicate pathology. The probable cause of the problem is supine hypotension. b. This is a normal finding that does not warrant immediate notification to her care provider. c. Correct. Teaching the woman to avoid lying on her back because of occlusion of the vena cava with the gravid uterus causes supine hypotension syndrome. d. Antenatal testing is not indicated with supine hypotension. KEY: Integrated Process: Clinical Problem Solving | Cognitive Level: Analysis | Content Area: Maternity | Client Need: Physiological Adaptation | Difficulty Level: Moderate

17. While performing Leopold's maneuvers on a woman in early labor, the nurse palpates a flat area in the fundal region, a hard round mass on the left side, a soft round mass on the right side, and small parts just above the symphysis. The nurse concludes which of the following? a. The fetal position is right occiput posterior. b. The fetal attitude is flexed. c. The fetal presentation is scapular. d. The fetal lie is vertical.

ANS: c Feedback a. This is a shoulder presentation. b. It is not possible to determine whether the attitude is flexed or not when doing Leopold's maneuvers. c. This is a shoulder presentation. d. The lie is transverse or horizontal. KEY: Integrated Process: Nursing Process: Analysis; Nursing Process: Assessment | Cognitive Level: Application | Content Area: Antepartum Care | Client Need: Health Promotion and Maintenance | Difficulty Level: Easy

A woman is 3 hours post-early-postpartum hemorrhage of 800 mL at delivery. Select the nursing actions for care of this patient. (Select all that apply.) a. Limit fluid intake to prevent nausea and vomiting. b. Assess fundus every 4 hours during the first 8 hours. c. Explain the importance of preventing an overdistended bladder. d. Provide assistance with ambulation.

ANS: c, d Fluid intake should be increased following a postpartum hemorrhage to decrease the risk of hypovolemia. The fundus should be assessed a minimum of every hour for the first 4 hours following a PPH. The woman needs to know the importance of preventing an overdistended bladder to decrease the risk of further hemorrhage. After postpartum hemorrhage, a woman is at risk for orthostatic hypotension.

37. The clinic nurse talks with Suzy, a pregnant woman at 9 weeks' gestation who has just learned of her pregnancy. Suzy's nausea and vomiting are most likely caused by (select all that apply): a. Increased levels of estrogen b. Increased levels of progesterone c. An altered carbohydrate metabolism d. Increased levels of human chorionic gonadotropin

ANS: c, d Nausea and vomiting during the first trimester most likely are related to rising levels of human chorionic gonadotropin (hCG) and altered carbohydrate metabolism. Changes in taste and smell, due to alterations in the oral and nasal mucosa, can further aggravate the gastrointestinal discomfort. KEY: Integrated Process: Nursing Process: Clinical Problem Solving | Cognitive Level: Application | Content Area: Maternity | Client Need: Safe and Effective Care Environment | Difficulty Level: Moderate

29. The perinatal nurse assessing a newborn for jaundice recalls that __________ is a process that converts the yellow lipid-soluble (nonexcretable) bilirubin pigment (present in bile) into a water-soluble (excretable) pigment.

ANS: conjugation Conjugation of bilirubin constitutes a major function of the newborn's liver. Conjugation is a process that converts the yellow lipid-soluble (nonexcretable) bilirubin pigment (present in bile) into a water-soluble (excretable) pigment.

8. The nurse is massaging a boggy uterus. The uterus does not respond to the massage. Which medication would the nurse expect would be given first: a. Methergine b. Ergotrate c. Carboprost d. Oxytocin or pitocin

ANS: d If the cause of the hemorrhage is uterine atony, continual fundal massage with lower uterine segment support is mandatory. While one member of the team massages the fundus, another nurse establishes intravenous access with a large bore needle and administers oxytocic drugs in the following order: oxytocin (Pitocin), followed by methylergonovine (Methergine) or ergonovine (Ergotrate), and carboprost (Hemabate).

11. The clinic nurse sees Xiao and her infant in the clinic for their 2-week follow-up visit. Xiao appears to be tired, her clothes and hair appear unwashed, and she does not make eye contact with her infant. She is carrying her son in the infant carrier and when asked to put him on the examining table, she holds him away from her body. The clinic nurse's most appropriate question to ask would be: a. "What has happened to you?" b. "Do you have help at home?" c. "Is there anything wrong with your son?" d. "Would you tell me about the first few days at home?"

ANS: d The well-baby checkup that generally takes place 1 to 2 weeks following the hospital discharge may offer the first opportunity to assess the mother-baby dyad. In this setting, the nurse needs to be alert for subtle cues from the new mother, such as making negative comments about the baby or herself, ignoring the baby's or other children's needs, as well as the mother's physical appearance. In a private area, the nurse should take time to explore the new mother's feelings. A nonthreatening way to open the dialogue might be to say: "Tell me how the first few days at home have gone." This statement provides the new mother with an opportunity to share both positive and negative impressions.

6. The following four babies are in the neonatal nursery. Which of the babies should be seen by the neonatologist as soon as possible? a. 1-day-old, HR 170 bpm, crying b. 2-day-old, T 98.9°F, slightly jaundice c. 3-day-old, breastfeeding q 2 h, rooting d. 4-day-old, RR 70 rpm, dusky coloring

ANS: d a. A slight tachycardia—170 bpm—is normal when a baby is crying. b. Slight jaundice on day 2 is within normal limits. c. It is normal for a breastfed baby to feed every 2 hours. d. A dusky skin color is abnormal in any neonate, whether or not the respiration rate is normal, although this baby is also slightly tachypneic.

If the umbilical cord prolapses during labor, the nurse should immediately: a. Type and cross-match blood for an emergency transfusion. b. Await MD order for preparation for an emergency cesarean section. c. Attempt to reposition the cord above the presenting part. d. Apply manual pressure to the presenting part to relieve pressure on the cord.

ANS: d Apply manual pressure to the presenting part to relieve pressure on the cord represents the first nursing intervention to attempt to improve circulation to the fetus.

A patient, G1 P0, is admitted to the labor and delivery unit for induction of labor. The following assessments were made on admission: Bishop score of 4, fetal heart rate 140s with good variability and no decelerations, TPR 98.6ºF, 88, 20, BP 120/80, negative obstetrical history. A prostaglandin suppository was inserted at that time. Which of the following findings, 6 hours after insertion, would warrant the removal of the Cervidil (dinoprostone)? a. Bishop score of 5 b. Fetal heart of 152 bpm c. Respiratory rate of 24 rpm d. Contraction frequency of every 2 minutes

ANS: d Cervidil should be removed for tachysystole.

Which of the following information regarding sexual activity would the nurse give a pregnant woman who is 35 weeks' gestation? a. Sexual activity should be avoided from now until 6 weeks postpartum. b. Sexual desire may be affected by nausea and fatigue. c. Sexual desire may be increased due to increased pelvic congestion. d. Sexual activity may require different positions to accommodate the woman's comfort.

ANS: d Correct. An enlarging abdomen creates feelings of awkwardness and bulkiness and may require couples to modify intercourse positions for the pregnant woman's comfort.

4. Blood volume expansion during pregnancy leads to: a. Iron-deficiency anemia b. Maternal iron stores being insufficient to meet the demands for iron in fetal development c. Plasma fibrin increase of 40% and fibrinogen increase of 50% d. Physiological anemia of pregnancy

ANS: d Feedback a. Iron-deficiency anemia is treated with iron supplementation. Iron-deficiency anemia is defined as hemoglobin of less than 11 g/dL and hematocrit less than 33%. b. Maternal iron stores that are insufficient to meet the demands for iron in fetal development result in iron-deficiency anemia. c. Hypercoagulation that occurs during pregnancy is to decrease the risk of postpartum hemorrhage. These changes taking place are not related to blood volume expansion. d. Correct. Physiological anemia of pregnancy, also referred to as pseudo-anemia of pregnancy, is due to hemodilution. The increase in plasma volume is relatively larger than the increase in RBCs that results in decreased hemoglobin and hematocrit values. KEY: Integrated Process: Clinical Problem Solving | Cognitive Level: Analysis | Content Area: Maternity | Client Need: Physiological Adaptation | Difficulty Level: Moderate

8. The clinic nurse talks to a 30-year-old woman at 34 weeks' gestation who complains of having difficulty sleeping. Jayne has noticed that getting back to sleep after she has been up at night is difficult. The nurse's best response is: a. "This is abnormal; it is important that you describe this problem to the doctor." b. "This is normal, and many women have this same problem during pregnancy; try napping for several hours each morning and afternoon." c. "This is abnormal; tell the doctor about this problem because diagnostic testing may be necessary." d. "This is normal in pregnancy, particularly during the third trimester when you also feel fetal movement at night; try napping once a day."

ANS: d Feedback a. This sleep pattern is a normal finding. b. Sleeping for several hours in the morning and afternoon would contribute to further sleep disturbances at night. c. This sleep pattern is a normal finding. d. Pregnancy sleep patterns are characterized by reduced sleep efficiency, fewer hours of night sleep, frequent awakenings, and difficulty going to sleep. Nurses can advise patients that afternoon napping may help alleviate the fatigue associated with the sleep alterations. KEY: Integrated Process: Teaching and Learning | Cognitive Level: Application | Content Area: Maternity | Client Need: Health Promotion and Maintenance | Difficulty Level: Moderate

Jenny, a 21-year-old single woman, comes for her first prenatal appointment at 31 weeks' gestation with her first pregnancy. The clinic nurse's most appropriate statement is: a. "Jenny, it is late in your pregnancy to be having your first appointment, but it is nice to meet you and I will try to help you get caught up in your care." b. "Jenny, have you had care in another clinic? I can't believe this is your first appointment!" c. "Jenny, by the date of your last menstrual period, you are 31 weeks and now that you are finally here, we need you to come monthly for the next two visits and then weekly." d. "Jenny, by your information, you are 31 weeks' gestation in this pregnancy. Do you have questions for me before I begin your prenatal history and information sharing?"

ANS: d The initial interview time with the patient should be used to build a positive, nonthreatening relationship and to gain her confidence by respecting her choices and advocating for continued prenatal care. The prenatal nurse's objective is to provide a user-friendly service that is efficient, effective, caring, and patient centered.

When providing a psychosocial assessment on a pregnant woman at 21 weeks' gestation, the nurse would expect to observe which of the following signs? a. Ambivalence b. Depression c. Anxiety d. Happiness

ANS: d The nurse would expect the patient to exhibit signs of happiness at this time.

The perinatal nurse understands that the most appropriate nursing action following an amniotomy is an assessment of the __________ as well as the __________ and __________ of the amniotic fluid.

ANS: fetal heart rate; color; odor The nurse carefully monitors the patient who will undergo an amniotomy. Vital signs, cervical effacement and dilation, station of the presenting part, fetal heart rate, and color and amount of amniotic fluid are assessed.

The perinatal nurse prepares for two potential complications that may accompany a precipitous labor and birth: postpartum __________ and a need for neonatal __________.

ANS: hemorrhage; resuscitation

The perinatal nurse provides information about postpartum depression to all families members because of the potential danger not only to the mother but also to the __________.

ANS: infant The earlier that postpartum depression is recognized and treatment begun, the better is the prognosis for a full recovery. The nurse should involve the family in helping the patient cope with her feelings and assisting with infant care.

The perinatal nurse caring for a laboring woman who is receiving an oxytocin infusion documents the following information: rate of __________, frequency and strength of __________, fetal __________, and cervical __________ and __________.

ANS: infusion; contractions; heart rate; dilatation; effacement Oxytocin protocols in many institutions require that the nurse remain at the patient's bedside at all times for careful surveillance. The following data should be placed on a flow sheet in the patient record: patient's vital signs, fetal heart rate, frequency, duration and strength of contractions, cervical effacement and dilatation, fetal station and lie, rate of oxytocin infusion intake and urine output, and the psychological response of the patient.

During labor, oxytocin is always administered __________.

ANS: intravenously with an infusion pumpDuring labor, oxytocin can only be administered intravenously via an infusion pump to titrate and regulate the dose for safe administration.

30. Providing information to parents about jaundice constitutes an important component of the nurse's discharge teaching. Ensuring that parents know when and who to call if their infant develops signs of jaundice will help decrease the risk of __________, or permanent brain damage.

ANS: kernicterus All newborns are screened before discharge for physiological jaundice. The central nervous system can be damaged from unconjugated bilirubin. If bilirubin crosses the blood-brain barrier, it can damage the cerebrum, causing a condition called kernicterus. Kernicterus occurs from brain cell necrosis and can permanently damage a newborn, depending on the amount of time the neurons are exposed to bilirubin, the susceptibility of the nervous system, and the function of the surviving neurons.

A nurse assesses a G2 P1 woman who gave birth to a 4500 gram baby boy 2 hours ago. The nurse notes that the woman's labor was only 2 hours and that the infant was delivered by the labor nurse. The nurse's assessment findings are: Fundus firm and midline at umbilicus Lochia heavy—saturates pad within 15 minutes and bleeding is a steady stream without clots Perineum intact, slight bruising Ice pack on perineum Vital signs are B/P 105/65, P 98, R 20, T 38° Based on this information, the nurse is concerned that the woman has a __________ of the __________ or __________.

ANS: laceration; cervix; vagina Based on the assessment data, the woman is experiencing an early postpartum hemorrhage (PPH). The hemorrhage is most likely not due to uterine atony because the fundus is firm and midline. Laceration of the cervix or vagina is the second most common cause of early PPH. This woman is displaying typical signs and symptoms of laceration of cervix or vagina—firm, midline fundus with steady stream of blood without clots.

33. Part of the assessment of a preterm infant includes obtaining an abdominal girth measurement. The NICU nurse performs this assessment because the patient is at risk for __________.

ANS: necrotizing enterocolitis (NEC) When caring for a child with necrotizing enterocolitis, the nurse must measure and record frequent abdominal circumferences, auscultate bowel sounds before every feeding, and observe the abdomen for distention (observable loops or shiny skin indicating distention).

79. The perinatal nurse knows that __________, which is the eating of nonnutritive substances, is a common __________.

ANS: pica; eating disorder Pica, the consumption of nonnutritive substances or food, is a common eating disorder that can affect pregnancy. Substances that are most often ingested include clay, dirt, cornstarch, and ice. KEY: Integrated Process: Nursing Process: Clinical Problem Solving | Cognitive Level: Knowledge | Content Area: Maternity | Client Need: Safe and Effective Care Environment | Difficulty Level: Easy

Postpartum woman are at an increased risk of thrombus formation immediately following birth due to an increased __________ level

ANS: plasma fibrinogen Levels of plasma fibrinogen tend to remain elevated during the first few postpartal weeks. Although this alteration exerts a protective effect against hemorrhage, it increases the patient's risk of thrombus formation

31. The NICU nurse recognizes that the infant who requires ventilation for meconium aspiration syndrome is most often __________.

ANS: post-term Meconium aspiration pneumonia occurs in 10% to 26% of all deliveries, and the incidence increases directly with gestational age. (Before 37 weeks' gestation there is a 2% incidence, and at 42 weeks' gestation there is a 44% incidence.)

72. The clinic nurse monitors the blood pressure and assesses a woman's urine at each prenatal visit to assess for signs or symptoms of __________. A previous history or the presence of a __________ are also risk factors.

ANS: preeclampsia; new partner A previous history of preeclampsia increases the woman's likelihood of a recurrence during subsequent pregnancies. If a woman did not experience preeclampsia with previous pregnancies but has a new partner for her current pregnancy, her risk of developing preeclampsia is similar to that of a woman who is pregnant for the first time. Although preeclampsia is a systemic disorder that occurs only during pregnancy, it is generally recognized by two classic symptoms: elevated blood pressure and proteinuria. KEY: Integrated Process: Nursing Process: Clinical Problem Solving | Cognitive Level: Application | Content Area: Maternity | Client Need: Safe and Effective Care Environment | Difficulty Level: Moderate

78. The clinic nurse understands that the physiological changes of pregnancy include vascular relaxation from the effects of __________ and impaired venous circulation from pressure exerted by the enlarged uterus, predisposing the pregnant woman to __________.

ANS: progesterone; varicose veins Progesterone results in vascular relaxation which combined with impaired venous return increases the incidence of varicose veins in pregnant women. KEY: Integrated Process: Nursing Process: Clinical Problem Solving | Cognitive Level: Knowledge | Content Area: Maternity | Client Need: Safe and Effective Care Environment | Difficulty Level: Easy

The perinatal nurse recognizes that the laboring multiparous patient who is attempting a vaginal birth following a previous cesarean birth (VBAC) needs frequent assessments to ensure that there is __________ during her labor.

ANS: progress Women with a previous history of cesarean birth may be offered a trial of labor, although a prompt cesarean birth is recommended at the earliest sign of maternal or fetal compromise.

69. During the prenatal class, the perinatal nurse describes factors that may initiate the process of labor. One of these factors is the production of __________, which are found in the uterine __________ and are released from the __________ at term as it softens and dilates.

ANS: prostaglandins; decidua or lining; cervix Prostaglandins are lipid substances found in high concentrations in the female reproductive tract and in the uterine decidua during pregnancy. Their exact function in pregnancy is unknown, although they may maintain a reduced placental vascular resistance. A decrease in prostaglandin levels may contribute to hypertension and preeclampsia. At term, an increased release of prostaglandins from the cervix as it softens and dilates may contribute to the onset of labor. KEY: Integrated Process: Teaching and Learning | Cognitive Level: Knowledge | Content Area: Maternity | Client Need: Health Promotion and Maintenance | Difficulty Level: Easy

A postpartum woman who describes symptoms of hallucinations and suicidal thoughts is most likely experiencing postpartum __________.

ANS: psychosis Postpartum psychosis is a rare but severe form of mental illness that severely affects not only the new mother, but the entire family. Postpartum psychosis may present with symptoms of postpartum depression. However, the distinguishing signs of psychosis are hallucinations, delusions, agitation, confusion, disorientation, sleep disturbances, suicidal and homicidal thoughts, and a loss of touch with reality.

76. The clinic nurse describes to the student nurse that __________ is excessive saliva production in pregnancy. This condition is most likely caused by increased __________ levels.

ANS: ptyalism; hormone Ptyalism, or excessive salivation, can be quite distressing for the pregnant woman who must frequently wipe her mouth or spit into a cup. Although the cause of ptyalism is unknown, it is most likely related to increased hormone levels. KEY: Integrated Process: Teaching and Learning | Cognitive Level: Application | Content Area: Maternity | Client Need: Safe and Effective Care Environment | Difficulty Level: Moderate

75. The prenatal nurse cautions a pregnant woman about Caesar salad consumption during pregnancy or any source of __________ or __________ milk.

ANS: raw eggs; unpasteurized A word of caution should be provided by health-care providers to pregnant women with regard to microbial food-borne illness. Raw, or unpasteurized, milk as well as partially cooked eggs and foods containing raw or partially cooked eggs should be avoided. KEY: Integrated Process: Teaching and Learning | Cognitive Level: Application | Content Area: Maternity | Client Need: Safe and Effective Care Environment | Difficulty Level: Moderate

74. The prenatal nurse describes the need for __________ and __________ screening at the first antenatal visit. If the pregnant woman is not immune, she will be counseled to avoid contact with young children who have a rash and could be infectious.

ANS: rubella; varicella Some of the routine maternal laboratory tests screen for childhood diseases that are known to cause congenital anomalies or other pregnancy complications if contracted during early pregnancy. When contracted during the first trimester, rubella causes a number of fetal deformities. Varicella (chickenpox) is another common childhood disease that may cause problems in the developing embryo and fetus. Therefore, all pregnant women are screened for rubella and varicella. KEY: Integrated Process: Teaching and Learning | Cognitive Level: Application | Content Area: Maternity | Client Need: Safe and Effective Care Environment | Difficulty Level: Moderate

77. The clinic nurse talks with the newly diagnosed pregnant woman about the nausea that the woman is experiencing in this pregnancy. The clinic nurse suggests eating __________ meals more often, remaining __________ after eating, and the using __________ techniques.

ANS: smaller; upright; relaxation Nausea is often one of the first symptoms of pregnancy experienced. Nurses can suggest strategies to help offset the nausea, such as the avoidance of "trigger foods" (foods that cause nausea from sight or smell) and tight clothing that constricts the abdomen. The use of relaxation techniques (i.e., slow, deep breathing, mental imagery) can also help to decrease nausea. Other techniques that are often helpful include consuming plain, dry crackers or sucking on peppermint candy before arising; adhering to small, frequent meals; and remaining in an upright position after eating. KEY: Integrated Process: Teaching and Learning | Cognitive Level: Application | Content Area: Maternity | Client Need: Safe and Effective Care Environment | Difficulty Level: Moderate

70. The perinatal nurse describes common complaints of pregnancy to the prenatal class attendees. Nasal __________, medically termed "__________ of pregnancy," is caused by increased levels of estrogen and progesterone.

ANS: stuffiness; rhinitis Nasal stuffiness and congestion (rhinitis of pregnancy) are common complaints during pregnancy. The nurse should educate the patient about these normal changes and offer reassurance. Increasing oral fluid intake helps to keep the mucus thin and easier to mobilize. KEY: Integrated Process: Teaching and Learning | Cognitive Level: Application | Content Area: Maternity | Client Need: Health Promotion and Maintenance | Difficulty Level: Moderate

The perinatal nurse explains to a new mother that the first sign of a postpartum infection will most likely be an increased __________.

ANS: temperature During the immediate postpartum period, the most common site of infection is the uterine endometrium. This infection presents with a temperature elevation over 101°F (38.4°C), often within the first 24 to 48 hours after childbirth, followed by uterine tenderness and foul-smelling lochia.

If a pregnant woman is group beta strep positive, prophylactic antibiotics should be administered if A. she is a planned c section B. the gestational age of her baby is less than 37 weeks C. she has vomiting and diarrhea during labor D. her baby has a known congenital anomaly

B. the gestational age of her baby is less than 37 weeks

Which of the following medications administered to the pregnant client with GDM and experiencing preterm labor requires close monitoring of the client's blood glucose levels? a. Nifedipine b. Betamethasone c. Magnesium sulfate d. Indomethacin

ANS: B Nifedipine does not affect maternal blood glucose levels. Beta-sympathomimetics may stimulate hyperglycemia which will require an increased need for insulin. Magnesium sulfate does not affect blood glucose levels. Indomethacin does not affect blood glucose levels.

Which of the following laboratory values is most concerning in a client with pregnancy-induced hypertension? a. Total urine protein of 200 mg/dL b. Total platelet count of 40,000 mm c. Uric acid level of 8 mg/dL d. Blood urea nitrogen 24 mg/dL

ANS: B The client's urine protein is elevated. A urine protein of ≥300 mg/dL in a 24-hour collection is considered concerning. Correct. A platelet count of ≤50,000 is a critical value and should be reported to the health-care provider immediately. This client is at increased risk of hemorrhage. The uric acid level is only slightly elevated. The BUN is only slightly elevated.

A woman at 10 weeks' gestation is diagnosed with gestational trophoblastic disease (hydatiform mole). Which of the following findings would the nurse expect to see? a. Platelet count of 550,000/ mm3 b. Dark brown vaginal bleeding c. White blood cell count 17,000/ mm3 d. Macular papular rash

ANS: B The nurse would not expect to see an elevated platelet count. The nurse would expect to see dark brown vaginal discharge The nurse would not expect to see an elevated white blood cell count. The nurse would not expect to see a rash.

2. Which of the following sites is priority for the nurse to assess when caring for a breastfeeding client, G8 P5, who is 1 hour postdelivery? a. Nipples b. Fundus c. Lungs d. Rectum

ANS: B a. Her nipples should be assessed, but this is not the priority assessment. b. This client is a grand multipara. She is high risk for uterine atony and postpartum hemorrhage. The nurse should monitor her fundus very carefully. c. Her lungs should be assessed bilaterally, but this is not the priority assessment. d. Her rectum should be assessed for hemorrhoids, but this is not the priority assessment.

80. Passive movement of the unengaged fetus

ANS: Ballottement Refer To: Glossary KEY: Integrated Process: Teaching and Learning | Cognitive Level: Knowledge | Content Area: Maternity | Client Need: Health Promotion and Maintenance | Difficulty Level: Easy

After an education class, the nurse overhears an adolescent woman discussing safe sex practices. Which of the following comments by the young woman indicates that additional teaching about sexually transmitted infection (STI) control issues is needed? a. "I could get an STI even if I just have oral sex." b. "Girls over 16 are less likely to get STDs than younger girls." c. "The best way to prevent an STI is to use a diaphragm." d. "Girls get human immunodeficiency virus (HIV) easier than boys do."

ANS: C This statement is true. Organisms that cause sexually transmitted infections can invade the respiratory and gastrointestinal tracts. This statement is true. Young women are especially high risk for becoming infected with sexually transmitted diseases. This statement is untrue. The young woman needs further teaching. Condoms protect against STDs and pregnancy. In addition, condoms can be kept in readiness for whenever sex may occur spontaneously. Using condoms does not require the teen to plan to have sex. A diaphragm is not an effective infection-control method. Plus, it would require the teen to plan for intercourse. This statement is true. Young women are higher risk for becoming infected with HIV than are young men.

15. A woman who is 12 weeks postpartum presents with the following behavior: she reports severe mood swings and hearing voices, believes her infant is going to die, she has to be reminded to shower and put on clean clothes, and she feels she is unable to care for her baby. These behaviors are associated with which of the following? a. Postpartum blues b. Postpartum depression c. Postpartum psychosis d. Maladaptive mother-infant attachment

ANS: C a. Postpartum blues usually occurs within the first few weeks of the postpartum period. Women experiencing postpartum blues will have mild mood swings, and they can take care of themselves as well as their baby. b. Women with PPD are predominately depressed and do not have mood swings. c. Postpartum psychosis is associated with a break from reality reflected in the woman hearing voices. d. The symptoms reported are reflective of a psychiatric disorder beyond maladaptive attachment.

16. The nurse is providing prenatal teaching to a group of diverse pregnant women. One woman, who indicates she smokes two to three cigarettes a day, asks about its impact on her pregnancy. The nurse explains that the most significant risk to the fetus is: a. Respiratory distress at birth b. Severe neonatal anemia c. Low neonatal birth weight d. Neonatal hyperbilirubinemia

ANS: C Feedback a. Respiratory distress is not the most significant risk to the fetus unless the fetus is also premature. b. Severe neonatal anemia is not associated with pregnancies complicated by cigarette smoking. c. Low neonatal birth weight is the most common complication seen in pregnancies complicated by cigarette smoking. d. Neonatal hyperbilirubinemia is not associated with pregnancies complicated by cigarette smoking. KEY: Integrated Process: Nursing Process: Implementation; Teaching and Learning | Cognitive Level: Application | Content Area: Antepartum Care; Growth and Development | Client Need: Health Promotion and Maintenance | Difficulty Level: Moderate

The perinatal nurse knows that tocolytic agents are most often used to (select all that apply): a. Prevent maternal infection b. Prolong pregnancy to 40 weeks' gestation c. Prolong pregnancy to facilitate administration of antenatal corticosteroids d. Allow for transport of the woman to a tertiary care facility

ANS: C, D Tocolytics are not used to treat maternal infection. Tocolytics are generally only effective in delaying delivery for several days. Presently, it is believed that the best reason to use tocolytic drugs is to allow an opportunity to begin the administration of antenatal corticosteroids to accelerate fetal lung maturity. Delaying the birth provides time for maternal transport to a facility equipped with a neonatal intensive care unit.

71. The clinic nurse promotes a diet rich in vitamin __________ during the third trimester to prevent the possibility of __________ rupture of the membranes.

ANS: C; premature Low levels of vitamin C may predispose women to premature rupture of membranes. As the cellular availability of vitamin C decreases, the rate of degradation of cervical collagen increases. With decreased collagen, the cervix more easily ripens, prompting effacement and dilatation. KEY: Integrated Process: Teaching and Learning | Cognitive Level: Application | Content Area: Maternity | Client Need: Health Promotion and Maintenance | Difficulty Level: Moderate

A 16-year-old patient is admitted to the hospital with a diagnosis of severe preeclampsia. The nurse must closely monitor the woman for which of the following? a. High leukocyte count b. Explosive diarrhea c. Fractured pelvis d. Low platelet count

ANS: D High leukocyte count is not associated with severe pregnancy-induced hypertension (PIH) or HELLP (hemolysis, elevated liver enzymes, and low platelets) syndrome. Explosive diarrhea is not associated with severe PIH or HELLP syndrome. A fractured pelvis is not associated with severe PIH or HELLP syndrome. Low platelet count is one of the signs associated with HELLP (hemolysis, elevated liver enzymes, and low platelets) syndrome.

The single most important risk factor for preterm birth includes: a. Uterine and cervical anomalies b. Infection c. Increased BMI d. Prior preterm birth

ANS: D The single most important factor is prior preterm birth with a reoccurrence rate of up to 40%.

62. Nosebleeds

ANS: Epistaxis Refer To: Glossary KEY: Integrated Process: Teaching and Learning | Cognitive Level: Knowledge | Content Area: Maternity | Client Need: Health Promotion and Maintenance | Difficulty Level: Easy

57. Lesions at the gum line that bleed easily

ANS: Epulis gravidarum Refer To: Glossary KEY: Integrated Process: Teaching and Learning | Cognitive Level: Knowledge | Content Area: Maternity | Client Need: Health Promotion and Maintenance | Difficulty Level: Easy

The development of a large hematoma can place the postpartum woman at risk for __________.

ANS: shock Upon examination of the perineal or vulvar areas, the nurse may notice discoloration and bulging of the tissue at the hematoma site. If touched, the patient complains of severe tenderness, and the clinician generally describes the tissue as "full." If the hematoma is large, signs of shock may be evident, and the patient may exhibit an absence of lochia and an inability to void.

Which of the following statements is true regarding hyperbilirubinemia? A. Jaundice covers the entire body in pathological jaundice versus only the face in physiological jaundice. B. Jaundice occurs within the first 24 hours post birth in pathological jaundice versus after 24 hours in physiological jaundice. C. Kernicterus only occurs in pathological jaundice. D. Jaundice begins to appear in term neonates when the bilirubin level is 3 mg/dL.

B. Jaundice occurs within the first 24 hours post birth in pathological jaundice versus after 24 hours in physiological jaundice.

Your patient is a 25 year old gravida 1 woman who is 2 hours postpartum. You note that her fundus is firm and midline. she is experiencing a steady stream of blood. The bed linen under her is soaked in blood. based on these findings and observations, you suspect that she is exhibiting signs/symptoms of a pph related to A. uterine atony B. laceration of the cervical or vaginal area C. retained placental tissue D. fibroids

B. Laceration of the cervical or vaginal area

Women who experience mastitis should be instructed to A. stop breastfeeding until 48 hours after the start of antibiotic treatment B. continue to breastfeed or massage and express milk from the affected breast. C. Wash nipples with antibiotic soap before each feeding D. apply cream to nipples after each feeding until resolved.

B. continue to breastfeed or massage and express milk from the affected breast.

You are assigned a woman who is 5 hours post birth. She gave birth to an 8 pound baby girl and experienced a 4th degree tear of the perineum. During assessment, she has informed you that she has rectal pressure and pain where she tore. Her level is a 10 on a pain scale. You note that her perineum is intact with minimal bruising. Her blood pressure is 100/60 and pulse is 98. Best action? A. Medicate her for pain B. Notify her physician of your data C. Place an ice pack on perineum D. Assist her in ambulating to the bathroom

B. notify her physician of data

When conducting a psychosocial assessment it is important to determine the ___________, ___________, & ___________ of mood or emotional disturbances. A. Anxiety, depression, level B. Coping, support, distress C. Frequency, duration, intensity D. Acceptance, avoidance, character

C. Frequency, duration, intensity

A neonate born at 37 weeks gestation is determined to be small for gestational age. The most common immediate problem for this infant would be A. anemia B. hypovolemia C. hypoglycemia D. Hypocalcemia

C. Hypoglycemia

When gavage feeding a preterm neonate, the nurse should A. measure the tube before insertion from the mouth to the sternum B. Check for placement by injecting a small amount of sterile water into the feeding tube and listen for a gurgling noise C. Instill formula over a 20 minute period of time D. Flush the tube at the end of feeding with dextrose water.

C. Instill formula over a 20 minute period of time

According to May's research, it is during the ____________ phase that many expectant fathers appear to put conscious thought of their partner's pregnancy aside. A. Announcement B. Transition C. Moratorium D. Developing

C. Moratorium

The primary risk factor for necrotizing enterocolitis is A. Early oral feedings with formula B. Passage of meconium during labor C. Prematurity D. Low birth weight

C. Prematurity

Foul-smelling lochia occurs A. when beta-hemolytic streptococcus is the primary organism associated with metritis B. Within the first 24 hours post birth related to metritis C. when the entire endometrium is infected. D. during the normal involution process

C. when the entire endometrium is infected

During a 6 week postpartum clinic visit, your patient tells you that she is concerned about her husband. She tells you that he either stays late at work or goes out after work with his friends. When he is home, he is usually drinking beer while watching sports on TV. What is the most appropriate response A. These are common behaviors of men as they proces the meaning of fatherhood. You just need to give him time to work through this life change. B. These behaviors can indicate that your husband does not want to take on the responsibilites of being a father. You need to talk to him about his feelings regarding fatherhood. C. These behaviors indicate that your husband is concerned about the added cost of having a newborn. You and your husband need to sit down and set up a budget. D. These behaviors might indicate that your husband is depressed. You need to encourage him to see a mental health professional.

D. These behaviors might indicate that your husband is depressed. You need to encourage him to see a mental health professional.

A common characteristic of a premature infant is A. Absence lanugo B. Dry skin C. Increased flexion of arms and legs D. Transparent and red skin

D. Transparent and red skin

You are assigned a woman who is 2 hours post birth. She had an emergency c section for an abruptio placenta. Base on this history, the woman is at risk for? A. Disseminated intravascular coagulation B. Retained placenta fragments C. Thrombosis D. Subinvolution of the uterus

A. DIC

Which is not a risk to the infant of a diabetic mother A. hyperglycemia B. poor feeding C. Marcosomia D. respiratory distress

A. hyperglycemia

9. Approximately 8 hours ago, Juanita, a 32-year-old G1 P0, gave birth after 2 ½ hours of pushing. She required an episiotomy and an assisted birth (forceps) due to the weight and size of her baby (9 lb. 9 oz.). The perinatal nurse is performing an assessment of Juanita's perineal area. A slight bulge is palpated and the presence of ecchymoses to the right of the episiotomy is noted. The area feels "full" and is approximately 4 cm in diameter. Juanita describes this area as "very tender." The most likely cause of these signs and symptoms is: a. Hematoma formation b. Sepsis in the episiotomy site c. Inadequate repair of the episiotomy d. Postpartum hemorrhage

ANS: a A hematoma is a localized collection of blood in connective or soft tissue under the skin that follows injury of or laceration to a blood vessel without injury to the overlying tissue. The most common sign or symptom of a hematoma is unremitting pain and pressure. Upon examination of the perineal or vulvar areas, the nurse may notice discoloration and bulging of the tissue at the hematoma site. If touched, the patient complains of severe tenderness, and the clinician generally describes the tissue as "full."

13. The laboratory reported that the L/S ratio (lecithin/sphingomyelin) results from an amniocentesis of a gravid client with preeclampsia are 2:1. The nurse interprets the result as which of the following? a. The baby's lung fields are mature. b. The mother is high risk for hemorrhage. c. The baby's kidneys are functioning poorly. d. The mother is high risk for eclampsia

ANS: a a. An L/S ratio of 2:1 usually indicates that the fetal lungs are mature. b. L/S ratios are unrelated to maternal blood loss. c. L/S ratios are unrelated to fetal renal function. d. L/S ratios are unrelated to maternal risk for becoming eclamptic.

12. A baby boy was just born to a mother who had positive vaginal cultures for group B streptococci. The mother was admitted to the labor room 30 minutes before the birth. For which of the following should the nursery nurse closely observe this baby? a. Grunting b. Acrocyanosis c. Pseudostrabismus d. Hydrocele

ANS: a a. This infant is high risk for respiratory distress. The nurse should observe this baby carefully for grunting. b. Acrocyanosis is a normal finding. c. Pseudostrabismus is a normal finding. d. Hydrocele should be reported to the neonatologist. It is not, however, an emergent problem, and it is not related to group B streptococci colonization in the mother.

A pregnant client at 20 weeks' gestation comes to the clinic for her prenatal visit. Which of the following client statements would indicate a need for further assessment? a. "I hate it when the baby moves." b. "I've started calling my mom every day." c. "My partner and I can't stop talking about the baby." d. "I still don't know much time I'm going to take off work after the baby comes."

ANS: a Experiencing quickening as unpleasant may be a sign of maladaptation to pregnancy and needs further assessment by the nurse.

Taboos are cultural restrictions that: a. Have serious supernatural consequences b. Have serious clinical consequences c. Have superstitious consequences d. Are functional and neutral practices

ANS: a Taboos are believed to have serious supernatural consequences. Taboos are not known to have clinical or superstitious consequences and are not functional or neutral practices.

The nurse is assisting a physician in the delivery of a baby via vacuum extraction. Which of the following nursing diagnoses for the gravida is appropriate at this time? a. Risk for injury b. Colonic constipation c. Risk for impaired parenting d. Ineffective individual coping

ANS: a There is a risk for injury. For example, the patient could suffer a cervical, vaginal, or perineal laceration.

A first-time father is experiencing couvade syndrome. He is likely to exhibit which of the following symptoms or behaviors? a. Urinary frequency b. Hypotension c. Bradycardia d. Prostatic hypertrophy

ANS: a Urinary frequency is a common symptom of couvade.

Which of the following actions can decrease the risk for a postpartum infection? (Select all that apply.) a. Diet high in protein and vitamin C b. Increased fluid intake c. Ambulating within a few hours after delivery d. Washing nipples with soap prior to each breastfeeding session

ANS: a, b, c Protein and vitamin C assist with tissue healing. Rehydrating a woman after delivery can assist with decreasing risk for infections. Early ambulation decreases risk for infection by promoting uterine drainage. The woman should not wash her breasts with soap because soap can dry the tissue and increase the woman's risk for tissue breakdown

Which of the following are primary risk factors for subinvolution of the uterus? (Select all that apply.) a. Fibroids b. Retained placental tissue c. Metritis d. Urinary tract infection

ANS: a, b, c Uterine fibroids can interfere with involution. Retained placental tissue does not allow the uterus to remain contracted. Infection in the uterus is a risk factor for subinvolution. UTI does not interfere with involution of the uterus.

49. Physiologic changes that occur in the renal system during pregnancy predispose the pregnant woman to urinary tract infections (UTIs). Symptoms of a UTI include (select all that apply): a. Dysuria b. Hematuria c. Urgency d. Delayed urination

ANS: a, b, c Urinary tract infection (UTI) symptoms include dysuria, hematuria, and urgency. KEY: Integrated Process: Knowledge | Cognitive Level: Complication | Content Area: Maternity | Client Need: Physiologic Adaptation | Difficulty Level: Moderate

23. Which of the following are common assessment findings of postmature neonates? (Select all that apply.) a. Dry and peeling skin b. Abundant vernix caseosa c. Hypoglycemia d. Thin, wasted appearance

ANS: a, b, c, d a. Vernix caseosa covers the fetus's body around 17 to 20 weeks' gestation; as pregnancy advances, the amount of vernix decreases. Vernix prevents water loss from the skin to the amniotic fluid; as the amount of vernix decreases, an increasing amount of water is lost from the skin. This contributes to the dry and peeling skin seen in postmature neonates. b. Vernix caseosa covers the fetus's body around 17 to 20 weeks' gestation; as pregnancy advances, the amount of vernix decreases. c. Placental insufficiency related to the aging of the placenta may result in postmaturity syndrome, in which the fetus begins to use its subcutaneous fat stores and glycemic stores. This results in the thin and wasted appearance of the neonate and risk for hypoglycemia during the first few hours post-birth. d. Placental insufficiency related to the aging of the placenta may result in postmaturity syndrome, in which the fetus begins to use its subcutaneous fat stores and glycemic stores. This results in the thin and wasted appearance of the neonate and risk for hypoglycemia during the first few hours post-birth.

55. During the initial antenatal visit, the clinic nurse asks questions about the woman's nutritional intake. Specific questions should include information pertaining to (select all that apply): a. Preferred foods b. The presence of cravings c. Use of herbal supplements d. Aversions to certain foods and odors

ANS: a, b, c, d The nurse should obtain a nutritional history on all pregnant patients and patients of childbearing age to gain specific information related to the pregnancy, including foods that are preferred while pregnant (which may provide information about cultural and environmental dietary factors), special diets (which will assist the nurse in planning for education or interventions for risk factors associated with dietary practices), cravings or aversions to specific foods, and use of herbal supplements. KEY: Integrated Process: Nursing Process: Clinical Problem Solving | Cognitive Level: Application | Content Area: Maternity | Client Need: Safe and Effective Care Environment | Difficulty Level: Moderate

48. Presumptive signs of pregnancy include (select all that apply): a. Nausea b. Fatigue c. Ballottement d. Amenorrhea

ANS: a, b, d Nausea and vomiting, fatigue, and amenorrhea are all common during pregnancy and are the presumptive signs of pregnancy. Ballottement is a probably sign, noted during a vaginal exam. KEY: Integrated Process: Nursing Process: Assessment | Cognitive Level: Knowledge | Content Area: Maternity | Client Need: Health Promotion and Maintenance | Difficulty Level: Moderate

33. An 18-year-old woman at 23 weeks' gestation tells the nurse that she has fainted two times. The nurse teaches about the warning signs that often precede syncope so that she can sit or lie down to prevent personal injury. Warning signs include (select all that apply): a. Sweating b. Nausea c. Chills d. Yawning

ANS: a, b, d Sweating is a warning sign that often precedes syncope. Syncope (a trandient loss of consciousness and postural tone with spontaneous recovery) during pregnancy is frequently attributed to orthostatic hypotension or inferior vena cava compression by the gravid uterus. Nausea and yawning are warning signs that often precede syncope. Lightheadedness, sweating, nausea, yawning, and feelings of warmth are warning signs that often precede syncope. Chills are not a warning sign that often precede syncope. KEY: Integrated Process: Nursing Process: Clinical Problem Solving | Cognitive Level: Application | Content Area: Maternity | Client Need: Safe and Effective Care Environment | Difficulty Level: Moderate

Nursing actions focused at reducing a postpartum woman's risk for cystitis include which of the following? (Select all that apply.) a. Voiding within a few hours post-birth b. Oral intake of a minimum of 1000 mL per day c. Changing peri-pads every 3 to 4 hours or more frequently as indicated d. Reminding the woman to void every 3 to 4 hours while awake

ANS: a, c, d Early voiding helps flush bacteria from the urethra. Voiding every 3 to 4 hours will decrease the risk of bacterial growth in the bladder. Soiled peri-pads are a media for bacterial growth. It is recommend that a postpartum woman drink a minimum of 3000 mL/day to help dilute urine and promote frequent voiding.

50. Urinary tract infection (UTI) prevention measures during pregnancy include counseling the pregnant woman to (select all that apply): a. Delay urination until bladder is full b. Limit hydration c. Wipe from front to back d. Urinate after intercourse

ANS: a, c, d Anticipatory guidance for urinary tract infection prevention includes delaying urination, wipe front to back, and maintaining adequate hydration. KEY: Integrated Process: Teaching and Learning | Cognitive Level: Knowledge | Content Area: Maternity | Client Need: Health Promotion and Maintenance | Difficulty Level: Moderate

56. The perinatal nurse talks to the prenatal class attendees about guidelines for exercise in pregnancy. Recommended guidelines include (select all that apply): a. Stopping if the woman is tired b. Bouncing and slowly arching the back c. Increasing fluid intake throughout the physical activity d. Maintaining the ability to walk and talk during exercise

ANS: a, c, d Women should adhere to some basic safety guidelines when formulating their exercise program, including monitoring the breathing rate and ensuring that the ability to walk and talk comfortably is maintained during physical activity, stopping exercise when the woman becomes tired, and maintaining adequate fluid intake. Pregnant women should avoid exercises that can cause any degree of trauma to the abdomen or those that include rigorous bouncing, arching of the back, or bending beyond a 45-degree angle. KEY: Integrated Process: Teaching and Learning | Cognitive Level: Application | Content Area: Maternity | Client Need: Health Promotion and Maintenance | Difficulty Level: Moderate

19. Which of the following factors increases the risk of necrotizing enterocolitis (NEC) in very premature neonates? (Select all that apply.) a. Early oral feedings with formula b. Prolonged use of mechanical ventilation c. Hyperbilirubinemia d. Nasogastic feedings

ANS: a, d a. Preterm neonates have a decreased ability to digest and absorb formula. Undigested formula can cause a blockage in the intestines leading to necrosis of the bowel. b. Preterm neonates are predisposed to NEC due to alteration in blood flow to the intestines, impaired gastrointestinal host defense, and alteration in inflammatory response. c. Preterm neonates are predisposed to NEC due to alteration in blood flow to the intestines, impaired gastrointestinal host defense, and alteration in inflammatory response. d. Bacterial colonization in the intestines can occur from contaminated feeding tubes causing an inflammatory response in the bowel.

1. Folic acid supplementation during pregnancy is to: a. Improve the bone density of pregnant women b. Decrease the incidence of neural tube defects in the fetus c. Decrease the incidence of Down syndrome in the fetus d. Improve calcium uptake in pregnant women

ANS: b

7. Karen, a G2, P1, experienced a precipitous birth 90 minutes ago. Her infant is 4200 grams and a repair of a second-degree laceration was needed following the birth. As part of the nursing assessment, the nurse discovers that Karen's uterus is boggy. Furthermore, it is noted that Karen's vaginal bleeding has increased. The nurse's most appropriate first action is to: a. Assess vital signs including blood pressure and pulse. b. Massage the uterine fundus with continual lower segment support. c. Measure and document each perineal pad changed in order to assess blood loss. d. Ensure appropriate lighting for a perineal repair if it is needed.

ANS: b As the primary caregiver, the registered nurse may be the first person to identify excessive blood loss and initiate immediate actions. The nurse should first locate the uterine fundus and initiate fundal massage. Nursing actions performed after the massage are frequent vital sign measurements with an automatic device, measuring the length of time it takes for blood loss to saturate a pad, and assessing for bladder distention.

3. A full-term neonate who is 30 hours old has a bilirubin level of 10 mg/dL. The neonate has a yellowish tint to the skin of the face. The mother is breastfeeding her newborn. The nurse caring for this neonate would anticipate which of the following interventions? a. Phototherapy b. Feeding neonate every 2 to 3 hours c. Switch from breastfeeding to bottle feeding d. Assess red blood cell count

ANS: b a. Phototherapy is considered when the levels are 12 mg/dL or higher when the neonate is 25 to 48 hours old. Neonates re-absorb increased amounts of unconjugated bilirubin in the intestines due to lack of intestinal bacteria and decreased gastrointestinal motility. b. Adequate hydration promotes excretion of bilirubin in the urine. c. Colostrum acts as a laxative and assists in the passage of meconium. d. Assessing RBC is not a treatment for hyperbilirubinemia.

A woman presents for prenatal care at 6 weeks' gestation by LMP. Which of the following findings would the nurse expect to see? a. Multiple pillow orthopnea b. Maternal ambivalence c. Fundus at the umbilicus d. Pedal and ankle edema

ANS: b Ambivalence is a common feeling of women during the first trimester.

Jane's husband Brian has begun to put on weight. What is this a possible sign of? a. Culturalism syndrome b. Couvade syndrome c. Moratorium phase d. Attachment

ANS: b Correct. Couvade syndrome has symptoms that mimic changes of pregnancy.

28. A baby has just been admitted into the neonatal intensive care unit with a diagnosis of intrauterine growth restriction (IUGR). Which of the following maternal problems could have resulted in this complication? (Select all that apply.) a. Cholecystitis b. Hypertension c. Cigarette smoker d. Candidiasis e. Cerebral palsy

ANS: b, c Babies born to women with cholecystitis are not especially high risk for IUGR. Babies born to women with PIH or who smoke are high risk for IUGR. Babies born to women with candidiasis or cerebral palsy are not especially high risk for IUGR.

4. The perinatal nurse accurately defines postpartum hemorrhage by including a decrease in hematocrit levels from pre- to postbirth by: a. 5% b. 8% c. 10% d. 15%

ANS: c Historically, practitioners have defined postpartum hemorrhage as a blood loss greater than 500 mL following a vaginal birth and 1000 mL or more following a cesarean birth. Hematocrit levels that decrease 10% from pre- to postbirth measurements are also included in the definition.

11. A 1-day-old neonate in the well-baby nursery is suspected of suffering from drug withdrawal because he is markedly hyperreflexic and is exhibiting which of the following additional sign or symptom? a. Prolonged periods of sleep b. Hypovolemic anemia c. Repeated bouts of diarrhea d. Pronounced pustular rash

ANS: c a. Babies who are withdrawing from drugs have disorganized behavioral states and sleep very poorly. b. There is nothing in the scenario that indicates that this child is hypovolemic or anemic. c. Babies who are experiencing withdrawal often experience bouts of diarrhea. d. A pustular rash is characteristic of an infectious problem, not of neonatal abstinence syndrome.

4. A NICU nurse is caring for a full-term neonate being treated for group B streptococcus. The mother of the neonate is crying and shares that she cannot understand how her baby became infected. The best response by the nurse is: a. "Newborns are more susceptible to infections due to an immature immune system. Would you like additional information on the newborn immune system?" b. "The infection was transmitted to your baby during the birthing process. Do you have a history of sexual transmitted infections?" c. "Approximately 10% to 30% of women are asymptomatic carries of group B streptococcus which is found in the vaginal area. What other questions do you have regarding your baby's health?" d. "I see that this is very upsetting for you. I will come back later and answer your questions."

ANS: c a. Correct information, but does not fully address the woman's concern. b. Correct, but GBS is not a sexually transmitted disease. c. Correct. This response answers her questions and allows her to ask additional questions about her baby's health. d. Acknowledges that she is upset but does not provide immediate information.

7. A multipara, 26 weeks' gestation and accompanied by her husband, has just delivered a fetal demise. Which of the following nursing actions is appropriate at this time? a. Encourage the parents to pray for the baby's soul. b. Advise the parents that it is better for the baby to have died than to have had to live with a defect. c. Encourage the parents to hold the baby. d. Advise the parents to refrain from discussing the baby's death with their other children.

ANS: c a. It is inappropriate for the nurse to advise prayer. The parents must decide for themselves how they wish to express their spirituality. b. This is an inappropriate suggestion. c. This is an appropriate suggestion. Encouraging parents to spend time with their baby and hold their baby is an action that supports the parents during the grieving process. d. This is an inappropriate suggestion. It is very important for the parents to clearly communicate the baby's death with their other children.

16. The NICU nurse recognizes that respiratory distress syndrome results from a developmental lack of: a. Lecithin b. Calcium c. Surfactant d. Magnesium

ANS: c a. The ratio of lecithin to sphingomyelin in the amniotic fluid is used to assess maturity of fetal lungs. b. Calcium is needed to prevent undermineralization of bones. c. Respiratory distress syndrome (RDS) is a developmental respiratory disorder that affects preterm newborns due to lack of lung surfactant. The pathology of RDS is that there is diffuse atelectasis with congestion and edema in the lung spaces. On deflation, the alveoli collapse, and there is decreased lung compliance. d. Magnesium is needed to prevent undermineralization of bones.

9. It is noted that the amniotic fluid of a 42-week gestation baby, born 30 seconds ago, is thick and green. Which of the following actions by the nurse is critical at this time? a. Perform a gavage feeding immediately. b. Assess the brachial pulse. c. Assist a physician with intubation. d. Stimulate the baby to cry.

ANS: c a. This action is not appropriate. The baby needs tracheal suctioning. b. The baby needs to have tracheal suctioning. The most important action to promote health for the baby is for the health-care team to establish an airway that is free of meconium. c. This action is appropriate. The baby needs to be intubated in order for deep suctioning to be performed by the physician. A nurse would not intubate and suction but rather would assist with the procedures. d. It is strictly contraindicated to stimulate the baby to cry until the trachea has been suctioned. The baby would aspirate the meconium-stained fluid, which could result in meconium-aspiration syndrome.

What is the most common expected emotional reaction of a woman to the news that she is pregnant? a. Jealousy b. Acceptance c. Ambivalence d. Depression

ANS: c Ambivalence is a normal expected reaction to the news of pregnancy, whether or not the pregnancy is planned or wanted.

21. Nursing actions that minimize oxygen demands in the neonate include which of the following? (Select all that apply.) a. Providing frequent rest breaks when feeding b. Placing neonate on back for sleeping c. Maintaining a neutral thermal environment (NTE) d. Clustering nursing care

ANS: c, d a. A prolonged feeding session increases energy consumption that increases oxygen consumption. b. Placing the neonate on the back for sleeping has no effect on oxygen consumption. c. A decrease in environmental temperature leads to a decrease in the neonate's body temperature which leads to an increase in respiratory and heart rate that leads to an increase in oxygen consumption. d. Clustering of nursing care decreases stress which decreases oxygen requirements.

46. The clinic nurse explains to Margaret, a newly diagnosed pregnant woman at 10 weeks' gestation, that her rubella titer indicates that she is not immune. Margaret should be advised to (select all that apply): a. Avoid contact with all children b. Be retested in 3 months c. Receive the rubella vaccine postpartum d. Report signs or symptoms of fever, runny nose, and generalized red rash to the health-care provider

ANS: c, d Testing for rubella (German measles) is not necessary as titers are reliable indicators of immunity. Rubella (German measles) is one of the most commonly recognized viral infections known to cause congenital problems. If a woman contracts rubella during the first 12 weeks of pregnancy, the fetus has a 90% chance of being adversely affected. A maternity patient who is not immune to rubella should be offered the rubella immunization following childbirth, ideally prior to hospital discharge. The patient should report signs or symptoms of rubella during pregnancy to her health-care provider. It is not realistic for a woman to avoid contact with all children. KEY: Integrated Process: Teaching and Learning | Cognitive Level: Application | Content Area: Maternity | Client Need: Safe and Effective Care Environment | Difficulty Level: Moderate

26. A baby was born 4 days ago at 34 weeks' gestation. She is receiving phototherapy as ordered by the physician for physiological jaundice. She has symptoms of temperature instability, dry skin, poor feeding, lethargy, and irritability. The nurse's priority nursing action(s) is (are) to (select all that apply): a. Verify laboratory results to check for hypomagnesia. b. Verify laboratory results to check for hypoglycemia. c. Monitor the baby's temperature to check for hypothermia. d. Calculate 24-hour intake and output to check for dehydration

ANS: c, d There are two priority nursing interventions for hyperbilirubinemia. Hydration status is important if the newborn shows signs of dehydration such as dry skin and mucus membranes, poor intake, concentrated urine or limited urine output, and irritability. The newborn should also be kept warm while receiving phototherapy. When an infant is under phototherapy, the temperature needs to be monitored closely because the lights give off extra heat, but if the newborn is in an open crib and undressed, hypothermia may occur. Hypomagnesia and hypoglycemia are not related to phototherapy.

2. The positive signs of pregnancy are: a. All physiological and anatomical changes of pregnancy b. All subjective signs of pregnancy c. All those physiological changes perceived by the woman herself d. The objective signs of pregnancy that can only be attributed to the fetus

ANS: d

2. A nurse assesses that a 3-day-old neonate who was born at 34 weeks' gestation has abdominal distention and vomiting. These assessment findings are most likely related to: a. Respiratory Distress Syndrome (RDS) b. Bronchopulmonary Dysplasia (BPD) c. Periventricular Hemorrhage (PVH) d. Necrotizing Enterocolitis (NEC)

ANS: d a. Assessment findings for RDS include tachypnea, intercostal retractions, respiratory grunting, and nasal flaring. b. Assessment findings for BPD include chest retractions; audible wheezing, rales, and rhonchi; hypoxia; and bronchospasm. c. Assessment findings for PVH include bradycardia, hypotonia, full and/or tense anterior fontanel, and hyperglycemia. d. Assessment findings related to NEC include abdominal distention, bloody stools, abdominal distention, vomiting, and increased gastric residual. These signs and symptoms are related to the premature neonate's inability to fully digest stomach contents and limitation in absorptive function.

5. A nursery nurse observes that a full-term AGA neonate has nasal congestion, hypertonia, and tremors and is extremely irritable. Based on these observations, the nurse suspects which of the following? a. Hypoglycemia b. Hypercalcemia c. Cold stress d. Neonatal withdrawal

ANS: d a. Signs and symptoms of hypoglycemia are jitteriness, hypotonia, irritability, apnea, lethargy, and temperature instability, but not nasal congestion. b. Signs and symptoms of hypercalcemia are vomiting, constipation, and cardiac arrhythmias. c. Signs and symptoms of cold stress are decreased temperature, cool skin, lethargy, pallor, tachypnea, hypotonia, jitteriness, weak cry, and grunting. d. These are common signs and symptoms of neonatal withdrawal.

17. The NICU nurse is providing care to a 35-week-old infant who has been in the neonatal intensive care unit for the past 3 weeks. His mother wants to breastfeed her son naturally but is currently pumping her breasts to obtain milk. His mother is concerned that she is only producing about 1 ounce of milk every 3 hours. The nurse's best response to the patient's mother would be: a. "Pumping is hard work and you are doing very well. It is good to get about 1 ounce of milk every 3 hours." b. "Natural breastfeeding will be a challenging goal for your baby. Beginning today, you will need to begin to pump your breasts more often." c. "Your baby will not be ready to go home for at least another week. You can begin to pump more often in the next few days in preparation for taking your child home." d. "You have been working hard to give your son your breast milk. We can map out a schedule to help you begin today to pump more often to prepare to take your baby home."

ANS: d a. This is correct information but does not assist the women in producing more milk. b. This does not provide her with a plan to increase her milk. c. This does not provide her with a plan. d. The mother should be praised for her efforts to breastfeed and encouraged to continue to pump her milk. A determined schedule for pumping the milk will help the mother keep her milk flow steady and provide enough nutrients for the infant after discharge.

68. The clinic nurse explains to the new nurse that during pregnancy, the maternal metabolism is altered to support the pregnancy by the hormones __________ and __________, which are produced by the anterior __________ gland.

ANS: thyrotropin; adrenotropin; pituitary Maternal metabolism is altered to support the pregnancy by thyrotropin and adrenotropin. These hormones, produced by the anterior pituitary gland, exert their effects on the thyroid and adrenal glands. Thyrotropin causes an increased basal metabolism, and adrenotropin alters adrenal gland function to increase fluid retention by the kidneys. KEY: Integrated Process: Teaching and Learning | Cognitive Level: Knowledge | Content Area: Maternity | Client Need: Health Promotion and Maintenance | Difficulty Level: Easy

32. The NICU nurse's patient assignment includes an infant who is 25 weeks' gestation. The nurse knows that according to the gestational age, this infant would be described as __________.

ANS: very premature The definition of very premature is a neonate born at less than 32 weeks' gestation. The definition of premature is a neonate born between 32 and 34 weeks' gestation. The definition of late premature is a neonate born between 34 and 37 weeks' gestation.

To facilitate adaptation to pregnancy during the 1st trimester, an appropriate nursing action would be to: A. Encourage the woman to sign up for prepared childbirth classes B. Assess the woman's response to pregnancy C. Encourage the woman to "tune-in" to fetal movement D. Give anticipatory guidance regarding breastfeeding

B. Assess the woman's response to pregnancy

While planning care for the lesbian client, it is most important for the nurse to include a goal that addresses the need for: A. Referrals to a perinatal social worker on admission to the hospital B. Establishing a trusting relationship with HCP's C. Providing counseling to lesbian mothers regarding legal issues they may face with parenting D. Choosing a name for the infant and obtaining supplies for the new baby

B. Establishing a trusting relationship with HCP's

Prescriptive behavior is: A. Behavior during the childbearing period that is limited for pregnant women B. Expected behavior for pregnant women C. Behavior that is restricted for pregnant women that has supernatural consequences D. An unacceptable practice that has implications for pregnant women

B. Expected behavior for pregnant women

Clinical management strategies for prevention of retinopathy of prematurity focus on targeting appropriate ______ranges for infants at risk A. Arterial pH B. Oxygen saturation C. Heart rate D. Core temperature

B. Oxygen saturation

Nursing interventions that facilitate adaptation to pregnancy in the 1st trimester focus on: A. Physiological changes in pregnancy B. Promoting pregnancy and birth as a family experience C. Readiness for parenting D. Partners' role during labor and child birth

B. Promoting pregnancy and birth as a family experience

Ambivalent feelings toward pregnancy in the 3rd trimester may include: A. Normal expected findings B. Unresolved conflict C. Depression D. Unwanted pregnancy

B. Unresolved conflict


संबंधित स्टडी सेट्स

Cultural Anthropology Final Exam Ch. 6-10

View Set

FIN 3400 Finances Chapter 2-3 Financial Statements

View Set

4@512%sH@#....2.3.1.2.3.rTwsDfChapter 1 WEB DEVELOPEMENT FOR PERSONAL USE

View Set

Missed QBank Questions - Domain 2

View Set

Final Exam - Florida Life and Health

View Set

Introduction to Socket Programming

View Set